Уравнения по комбинаторике с решениями

Математика — онлайн помощь

Уравнения по комбинаторике с решениями

Рассмотрим множество, состоящее из n различных элементов. Требуется выбрать из них какие-нибудь k элементов и расположить эти k элементов в каком-либо порядке. Такие упорядоченные последовательности называются размещениями из n элементов по k элементов (упорядоченные – следовательно, последовательности и — различные размещения).

Если в последовательности нет одинаковых элементов, то говорят о размещении без повторений. Их количество

Уравнения по комбинаторике с решениями

Если в последовательности допускается наличие одинаковых элементов, то говорят о размещении с повторениями. Их количество

Уравнения по комбинаторике с решениями

Любое подмножество (неупорядоченное), состоящее из k элементов, называется сочетанием из n элементов по k элементов.

Различные сочетания отличаются друг от друга только самими входящими в них элементами, порядок их следования безразличен, т.е. по условию задачи подмножества и не различны (соединены).

Число сочетаний без повторений

Уравнения по комбинаторике с решениями.

Число сочетаний с повторениями

Уравнения по комбинаторике с решениями.

Количество способов переставить элементов в заданном множестве (количество перестановок) вычисляется по формуле

Уравнения по комбинаторике с решениями.

При решении простейших комбинаторных задач можно использовать следующую таблицу, определяющую число множеств, состоящих из k элементов, отбираемых из множества, содержащего n элементов

ВыборНеупорядоченныйУпорядоченный
Без повтораУравнения по комбинаторике с решениямиУравнения по комбинаторике с решениями
С повторомУравнения по комбинаторике с решениямиУравнения по комбинаторике с решениями

Рассмотрим разницу между сочетаниями, размещениями с повторениями, без повторений на следующих примерах.

Содержание
  1. ПРИМЕРЫ РЕШЕНИЯ ЗАДАЧ
  2. Примеры и задачи для самостоятельного решения
  3. Комбинаторика — правила, формулы и примеры с решением
  4. Всё о комбинаторике
  5. Комбинаторные задачи с решением
  6. Пример №1
  7. Пример №2
  8. Пример №3
  9. Пример №4
  10. Пример №5
  11. Пример №6
  12. Пример №7
  13. Пример №8
  14. Пример №9
  15. Пример №10
  16. Пример №11
  17. Пример №12
  18. Пример №13
  19. Пример №14
  20. Пример №15
  21. Пример №16
  22. Правила суммы и произведения
  23. Пример №17
  24. Пример №18
  25. Пример №19
  26. Пример №20
  27. Пример №21
  28. Пример №22
  29. Пример №23
  30. Размещения и перестановки
  31. Пример №24
  32. Пример №25
  33. Пример №26
  34. Пример №27
  35. Пример №28
  36. Пример №29
  37. Пример №30
  38. Пример №31
  39. Комбинации и бином ньютона
  40. Пример №32
  41. Пример №33
  42. Пример №34
  43. Пример №35
  44. Пример №36
  45. Пример №37
  46. Пример №38
  47. Пример №39
  48. Элементы комбинаторики
  49. Арифметика случайных событий
  50. Пример №40
  51. Теорема сложения вероятностей несовместных событий
  52. Зависимые и независимые события. Условная и безусловная вероятности
  53. Пример №41
  54. Теорема умножения вероятностей
  55. Что такое комбинаторика
  56. Понятие множества
  57. Равенство множеств
  58. Подмножество
  59. Операции над множествами
  60. Комбинаторика и Бином Ньютона
  61. Схема решения комбинаторных задач
  62. Понятие соединения
  63. Правило суммы
  64. Правило произведения
  65. Упорядоченные множества
  66. Размещения
  67. Пример №42
  68. Пример №43
  69. Пример №44
  70. Пример №45
  71. Перестановки
  72. Пример №46
  73. Пример №47
  74. Пример №48
  75. Сочетания без повторений
  76. Вычисление числа сочетаний без повторений с помощью треугольника Паскаля
  77. Пример №49
  78. Пример №50
  79. Бином Ньютона
  80. Объяснение и обоснование Бинома Ньютона
  81. Свойства биномиальных коэффициентов
  82. Пример №51
  83. Пример №52
  84. Зачем нужна комбинаторика
  85. Правило суммы
  86. Пример №53
  87. Правило произведения
  88. Пример №54
  89. Пример №55
  90. Пример №56
  91. Пример №57
  92. Пример №58
  93. Пример №59
  94. Пример №60
  95. Конспект урока на тему «Решение комбинаторных уравнений» (10 класс)
  96. 💡 Видео

Видео:Комбинаторика: перестановка, размещение и сочетание | Математика | TutorOnlineСкачать

Комбинаторика: перестановка, размещение и сочетание | Математика | TutorOnline

ПРИМЕРЫ РЕШЕНИЯ ЗАДАЧ

ПРИМЕР 13.2.1 В коробке 6 шаров, пронумерованных от 1 до 6. Из коробки вынимаются друг за другом 3 шара и в этом же порядке записывают полученные цифры. Сколько трехзначных чисел можно таким образом записать?

Решение: По условию задачи подмножества и – различные. Повторов в подмножестве быть не может, так как шары не возвращаются в коробку. Уравнения по комбинаторике с решениями

Уравнения по комбинаторике с решениями.

ПРИМЕР 13.2.2. В коробке 6 шаров пронумерованных от 1 до 6. Из коробки вынимаются 3 шара и записывают число в порядке возрастания цифр. Сколько трехзначных чисел можно таким образом записать?

Решение: По условию задачи подмножества и дают число 123, т.е. не являются различными.

Уравнения по комбинаторике с решениями.

ПРИМЕР 13.2.3. Условие задачи 2.1 (шары возвращаются в коробку)

Решение: Уравнения по комбинаторике с решениями.

ПРИМЕР 13.2.4. Условие задачи 2.2 (шары возвращаются в коробку)

Решение: Уравнения по комбинаторике с решениями.

ПРИМЕР 13.2.5. Сколько различных перестановок можно составить из букв слова «комар»?

Решение: Уравнения по комбинаторике с решениями.

ПРИМЕР 13.2.6. Сколько различных перестановок можно составить из букв слова «задача»?

Решение: Если бы все шесть букв слова были различны, то число перестановок было бы 6! Но буква «а» встречается в данном слове три раза, и перестановки только этих трех букв «а» не дают новых способов расположения букв. Поэтому число перестановок букв слова «задача» будет не 6!, а в 3! раза меньше, то есть Уравнения по комбинаторике с решениями.

ПРИМЕР 13.2.7. В мастерской имеется материал 5 цветов. Поступил заказ на пошив флагов, состоящих из трех горизонтальных полос разного цвета каждый. Сколько таких различных флагов может сшить мастерская?

Уравнения по комбинаторике с решениями.

Решение: Флаги отличаются друг от друга как цветом полос, так и их порядком, поэтому разных флагов можно сделать Уравнения по комбинаторике с решениямиштук.

ПРИМЕР 13.2.8. Сколькими способами можно распределить 5 учеников по 3 параллельным классам?

Решение: Составим вспомогательную таблицу

Номер ученика
Вариант класса

Таким образом, видно, что если для одного ученика существует 3 варианта выбора класса, то для всех 5 учеников существует Уравнения по комбинаторике с решениямиспособов распределения по классам.

ПРИМЕР 13.2.9. На книжной полке помещается 30 томов. Сколькими способами их можно расставить, чтобы при этом первый и второй том не стояли рядом?

Решение: Произведем рассуждения “от обратного”. Тридцать томов на одной полке можно разместить 30! способами.

Уравнения по комбинаторике с решениями.

Если 1 и 2 тома должны стоять рядом, то число вариантов расстановки сокращается до Уравнения по комбинаторике с решениями, т.к. комбинацию из 1 и 2 тома можно считать за один том, но при этом они могут стоять как (1;2) или (2;1), т.е.

Уравнения по комбинаторике с решениями, Уравнения по комбинаторике с решениями.

Тогда искомое число способов расстановки есть

Уравнения по комбинаторике с решениями

ПРИМЕР 13.2.10. Чемпионат, в котором участвуют 16 команд, проводится в два круга, т.е. каждая команда дважды встречается с любой другой. Определить, какое количество встреч следует провести.

Решение: По условию задачи из 16 команд для каждой встречи требуется отобрать 2 команды. В данном случае отбор производится без повтора и порядок отбора не важен, т.е. число вариантов — Уравнения по комбинаторике с решениями. Так как команды должны играть дважды число вариантов удваивается, т.е. Уравнения по комбинаторике с решениями.

ПРИМЕР 13.2.11. Автомобильная мастерская имеет для окраски 10 основных цветов. Сколькими способами можно окрасить автомобиль, если смешивать от 3 до 7 основных цветов?

Решение: По условию задачи отбор цветов для окраски производится без повтора и порядок отбора не важен, т.е. число вариантов зависит лишь от числа отбираемых для окраски цветов — Уравнения по комбинаторике с решениями. Поэтому общее число вариантов есть

Уравнения по комбинаторике с решениями.

ПРИМЕР 13.2.12. Турист прошел маршрут из пункта A в пункт B, из B в C и вернулся обратно. Сколько вариантов маршрута существует, если из пункта A в пункт B ведут 3 дороги, а из B в C — 4 и нельзя возвращаться той дорогой, по которой уже прошел?

Решение: Составим схему.

Уравнения по комбинаторике с решениями

Из рисунка видно, что вариантов маршрута из А в B существует 3, и из B в C – 4, т.е. всего маршрутов Уравнения по комбинаторике с решениями.

Уравнения по комбинаторике с решениями

На обратном пути вариантов маршрута из С в B существует 3 (один уже пройден), и из B в А – 2, т.е. всего возможных обратных маршрутов осталось Уравнения по комбинаторике с решениями. Тогда всего вариантов маршрута Уравнения по комбинаторике с решениями.

ПРИМЕР 13.2.13. Двенадцати ученикам выданы два варианта контрольной работы. Сколькими способами можно посадить учеников в два ряда по 6 человек, чтобы у сидящих рядом не было одинаковых вариантов, а у сидящих друг за другом был один и тот же вариант?

Решение: Рассуждения произведем несколькими способами

I способ) Первоначально 12 учеников разбивают на 2 группы по 6 человек. Это можно сделать Уравнения по комбинаторике с решениямиспособами.

Затем они могут распределиться по своим рядам согласно схеме

Уравнения по комбинаторике с решениями

Уравнения по комбинаторике с решениями.

Поэтому всего способов распределения учеников будет Уравнения по комбинаторике с решениями.

II способ) Первоначально 12 учеников запускают в класс, указывая место, где каждый должен сидеть, например “второй ряд, третье место”. Так как посадочных мест также 12, то всего вариантов распределения 12!
Варианты контрольной работы могут распределиться

“I вариант – I ряд, II вариант – II ряд”

“II вариант – I ряд, I вариант – II ряд”,

Таким образом, всего способов распределения учеников будет Уравнения по комбинаторике с решениями.

По приведенным решениям видно, что результаты решений совпадают.

ПРИМЕР 13.2.14. Сколько существует вариантов расположения шести гостей за круглым шестиместным столом?

Решение: Эта задача имеет разные решения и, соответственно разные ответы – в зависимости от того, что понимать под различным расположением гостей за столом. Поэтому исследуем возможные варианты.

Если считать, что нам важно, кто сидит на каком стуле, то это простая задача на перестановки и, следовательно, всего вариантов Уравнения по комбинаторике с решениями.

Если же важно не то, кто какой стул занял, а то, кто рядом с кем сидит, то требуется рассмотреть варианты взаимного расположения гостей. В таком случае, расположения гостей, получаемые одно из другого при повороте гостей вокруг стола, фактически являются одинаковыми (смотри рисунок).

Очевидно, что для любого расположения гостей таких одинаковых вариантов, получаемых друг из друга поворотом, — шесть. Тогда общее число вариантов уменьшается в шесть раз и их остается Уравнения по комбинаторике с решениями.
В случае же, когда нас интересует только взаимное расположение гостей, то одинаковыми можно считать и такие симметричные расположения, при которых у каждого гостя остаются те же соседи за столом, только левый и правый меняются местами (смотри рисунок).

В такой постановке вопроса общее число различных вариантов расположений гостей уменьшается вдвое и составляет 60.

Отметим, что каждое решение будет считаться правильным при соответствующей постановке задачи.

ПРИМЕР 13.2.15. Семнадцать студентов сдали экзамены по 4 предметам только на “хорошо” и “отлично”. Верно ли утверждение, что хотя бы у двух из них оценки по экзаменационным предметам совпадают?

Решение: Очевидно, что в данном случае речь идет о возможных вариантах вида

Предмет1234
Студент 14455
Студент 25445
Студент 35555
Студент 174454

Данный пример можно решить способом, изложенным в примере 13.1.8., и получить количество вариантов Уравнения по комбинаторике с решениями. Приведем другой наглядный способ решения, использующий так называемое “дерево решений”,который представляет все варианты (16 штук) получения экзаменационных оценок.

Уравнения по комбинаторике с решениями.

По “дереву решений” видно, что 16 студентов могут сдать экзамены только на “хорошо” и “отлично” так, что их результаты будут отличаться, но если студентов 17, хотя бы одно повторение обязательно будет.

При решении задач комбинаторики используются следующие правила.

Если некоторый объект A может быть выбран из совокупности объектов m способами, а другой объект B может быть выбран nспособами, то:

Правило суммы: выбрать либо A, либо B можно m+n способами.

Правило произведения. Пара объектов (A,B) в указанном порядке может быть выбрана Уравнения по комбинаторике с решениямиспособами.

Видео:Комбинаторное уравнениеСкачать

Комбинаторное уравнение

Примеры и задачи для самостоятельного решения

Решить комбинаторную задачу.

13.2.1.1. В группе 25 студентов. Сколькими способами можно выбрать старосту, заместителя старосты и профорга?

13.2.1.2. В группе 25 студентов. Сколькими способами можно выбрать актив группы, состоящий из старосты, заместителя старосты и профорга?

13.2.1.3. Сколькими способами можно составить список из 10 человек?

13.2.1.4. Сколькими способами из 15 рабочих можно создать бригады по 5 человек в каждой?

13.2.1.5. Буквы азбуки Морзе образуются как последовательности точек и тире. Сколько букв можно составить, используя для кодировки каждой из букв: а) ровно 5 символов? б) не более пяти символов?

13.2.1.6. Кости для игры в домино метятся двумя цифрами. Кости симметричны, и поэтому порядок чисел не существенен. Сколько различных костей можно образовать, используя числа 0,1,2,3,4,5,6?

13.2.1.7. Сколько различных звукосочетаний можно взять на десяти выбранных клавишах рояля, если каждое звукосочетание может содержать от трех до десяти различных звуков?

13.2.1.8. В вазе стоят 10 красных и 5 розовых гвоздик. Сколькими способами можно выбрать из вазы пять гвоздик одного цвета?

13.2.1.9. В некоторых странах номера трамвайных маршрутов обозначаются двумя цветными фонарями. Какое количество различных маршрутов можно обозначить, если использовать фонари восьми цветов?

13.2.1.10. Команда компьютера записывается в виде набора из восьми цифровых знаков – нулей и единиц. Каково максимальное количество различных команд?

13.2.1.11. Десять групп занимаются в десяти расположенных подряд аудиториях. Сколько существует вариантов расписания, при которых группы 1 и 2 находились бы в соседних аудиториях?

13.2.1.12. Два почтальона должны разнести 10 писем по 10 адресам. Сколькими способами они могут распределить работу?

13.2.1.13. Замок открывается только в том случае, если набран определенный трехзначный номер. Попытка состоит в том, что набирают наугад три цифры из заданных пяти. Угадать номер удалось только на последней из всех возможных попыток. Сколько попыток предшествовало удачной?

13.2.1.14. Номер автомобильного прицепа состоит из двух букв и четырех цифр. Сколько различных номеров можно составить, используя 30 букв и 10 цифр?

13.2.1.15. У одного студента есть 7 DVD дисков, а у другого – 9 дисков. Сколькими способами они могут обменять 3 диска одного на 3 диска другого?

13.2.1.16. На вершину горы ведут 7 дорог. Сколькими способами турист может два раза подняться на гору и спуститься с нее, если по одной и той же дороге нельзя проходить дважды?

13.2.1.17. У ювелира было 9 разных драгоценных камней: сапфир, рубин, топаз и т.д. Ювелир планировал изготовить браслет для часов, однако три камня было украдено. Насколько меньше вариантов браслета он может изготовить по сравнению с первоначальными планами?

13.2.1.18. В поезд метро на начальной станции вошли 10 пассажиров. Сколькими способами могут выйти все пассажиры на последующих 6 станциях?

13.2.1.19. За одним столом надо рассадить 5 мальчиков и 5 девочек так, чтобы не было двух рядом сидящих мальчиков и двух рядом сидящих девочек. Сколькими способами это можно сделать?

13.2.1.20. В классе 25 учеников. Верно ли утверждение, что, по крайней мере, у трех из них день рождения в один и тот же месяц?

13.2.1.21. На участке железной дороги расположено 25 станций с билетной кассой в каждой. Касса каждой станции продает билеты до любой другой станции, притом в обоих направлениях. Сколько различных вариантов билетов можно выдать на этом участке?

13.2.1.22. На официальном приеме 50 человек обменялись рукопожатиями. Сколько было сделано рукопожатий?

13.2.1.23. Сколько диагоналей у выпуклого двадцатиугольника?

Уважаемые студенты
На нашем сайте можно получить помощь по всем разделам математики и другим предметам:
✔ Решение задач
✔ Выполнение учебных работ
✔ Помощь на экзаменах

Видео:Решите уравнение ➜ ДВИ до ЕГЭСкачать

Решите уравнение ➜ ДВИ до ЕГЭ

Комбинаторика — правила, формулы и примеры с решением

Комбинаторика — это раздел математики, в котором изучаются способы выбора и размещения элементов некоторого конечного множества на основании определенных условий. Выбранные (или выбранные и размещенные) группы элементов называются соединениями. Если все элементы полученного множества разные, получаем соединения без повторений, а если элементы повторяются — соединения с повторениями.

Содержание:

В комбинаторике перестановка — это упорядоченный набор без повторений чисел.

Перестановкой из n элементов называется любое упорядоченное множество из n данных элементов.

Иными словами, это такое множество, для которого указано, какой элемент находится на первом месте, какой — на втором, . какой — на n-м.

Формула числа перестановок Уравнения по комбинаторике с решениями

Уравнения по комбинаторике с решениями

Количество различных шестизначных чисел, которые можно составить из цифр 1, 2, 3, 4, 5, 6, не повторяя эти цифры в одном числе, равноУравнения по комбинаторике с решениями

Размещением из n элементов по k называется любое упорядоченное множество из k элементов, состоящее из элементов данного n-элементного множества.

Формулы для нахождения количества соединений с повторениями обязательны только для классов физико-математического профиля.

Формула числа размещений Уравнения по комбинаторике с решениями

Уравнения по комбинаторике с решениями

Количество различных трехзначных чисел, которые можно составить из цифр 1, 2, 3, 4, 5, 6, если цифры не могут повторяться, равно

Уравнения по комбинаторике с решениями

Сочетанием без повторений из n элементов по k называется любое k-элементное подмножество данного n-элементного множества.

Формула числа сочетаний Уравнения по комбинаторике с решениями

Уравнения по комбинаторике с решениями(по определению считают, чтоУравнения по комбинаторике с решениями

Из 25 учащихся одного класса можно выделить пятерых для дежурства по школе Уравнения по комбинаторике с решениямиспособами, то есть Уравнения по комбинаторике с решениямиспособами.

Некоторые свойства числа сочетаний без повторений

Уравнения по комбинаторике с решениями(в частности, Уравнения по комбинаторике с решениями)

Уравнения по комбинаторике с решениями

Схема поиска плана решения простейших комбинаторных задач:

Если элемент А можно выбрать т способами, а элемент В — n способами (при этом выбор элемента А исключает одновременный выбор элемента В), то А или В можно выбрать m + n способами.

Если элемент А можно выбрать m способами, а после этого элемент В — n способами, то А и В можно выбрать Уравнения по комбинаторике с решениямиспособами.

Уравнения по комбинаторике с решениями

Объяснение и обоснование:

Понятие соединения. Правило суммы и произведения:

При решении многих практических задач приходится выбирать из определенной совокупности объектов элементы, имеющие те или иные свойства, размещать их в определенном порядке и т. д. Поскольку в этих задачах речь идет о тех или иных комбинациях объектов, то такие задачи называют комбинаторными. Раздел математики, в котором рассматриваются методы решения комбинаторных задач, называется комбинаторикой. В комбинаторике рассматривается выбор и размещение элементов некоторого конечного множества на основании определенных условий.

Выбранные (или выбранные и размещенные) группы элементов называют соединениями. Если все элементы полученного множества разные, получаем размещения без повторений, а если элементы могут повторяться — размещения с повторениями. В этом параграфе мы рассмотрим соединения без повторений.

Решение многих комбинаторных задач базируется на двух основных правилах — правиле суммы и правиле произведения.

Правило суммы. Если на тарелке лежат 5 груш и 4 яблока, то выбрать один фрукт (грушу или яблоко) можно 9 способами (5 + 4 = 9). В общем виде справедливо такое утверждение:

  • если элемент А можно выбрать m способами, а элемент В — n способами (при этом выбор элемента А исключает одновременный выбор элемента В), то А или В можно выбрать m + n способами.

Уточним содержание этого правила, используя понятие множеств и операций над ними.

Пусть множество А состоит из m элементов, а множество В -из n элементов. Если множества А и В не пересекаются (то есть Уравнения по комбинаторике с решениями), то множество А Уравнения по комбинаторике с решениямиВ состоит изУравнения по комбинаторике с решениямиэлементов.

Правило произведения. Если в киоске продают ручки 5 видов и тетради 4 видов, то выбрать набор из ручки и тетради (то есть пару — ручка и тетрадь) можно 5æ4 = 20 способами (поскольку с каждой из 5 ручек можно взять любую из 4 тетрадей). В общем виде имеет место такое утверждение:

  • если элемент А можно выбрать m способами, а после этого элемент В — n способами, то А и В можно выбрать Уравнения по комбинаторике с решениямиспособами.

Это утверждение означает, что если для каждого из m элементов А можно взять в пару любой из n элементов В, то количество пар равно произведению Уравнения по комбинаторике с решениями.

В терминах множеств полученный результат можно сформулировать следующим образом. Если множество А состоит из т элементов, а множество В — из n элементов, то множество всех упорядоченных пар* (а; b), где первый элемент принадлежит множеству А (а ∈ А), а второй  множеству В (b ∈ В), состоит из Уравнения по комбинаторике с решениямиэлементов.

Повторяя приведенные рассуждения несколько раз (или, более строго, используя метод математической индукции), получаем, что правила суммы и произведения можно применять при выборе произвольного конечного количества элементов.

Упорядоченные множества:

При решении комбинаторных задач приходится рассматривать не только множества, в которых элементы можно записывать в любом порядке, но и так называемые упорядоченные множества. Для упорядоченных множеств существенным является порядок следования их элементов, то есть то, какой элемент записан на первом месте, какой на втором и т. д. В частности, если одни и те же элементы записать в разном порядке, то мы получим различные упорядоченные множества. Чтобы различить записи упорядоченного и неупорядоченного множеств, элементы упорядоченного множества часто записывают в круглых скобках, например (1; 2; 3) ≠ (1; 3; 2).

Рассматривая упорядоченные множества, следует учитывать, что одно и то же множество можно упорядочить по-разному. Например, множество из трех чисел можно упорядочить по возрастанию: (–5; 1; 3), по убыванию: (3; 1; –5), по возрастанию абсолютной величины числа: (1; 3; –5) и т. д.

* Множество всех упорядоченных пар (а; b), где первый элемент принадлежит множеству А (а ∈ А), а второй — множеству В (b ∈ В), называют декартовым произведением множеств А и В и обозначают А × В. Отметим, что декартово произведение В × А также состоит из m*n элементов.

Заметим следующее: для того чтобы задать конечное упорядоченное множество из n элементов, достаточно указать, какой элемент находится на первом месте, какой на втором, . какой на n-м.

Размещения:

Размещением из n элементов по k называется любое упорядоченное множество из k элементов, состоящее из элементов заданного n-элементного множества.

Например, из множества, содержащего три цифры , можно составить следующие размещения из двух элементов без повторений:

(1; 5), (1; 7), (5; 7), (5; 1), (7; 1), (7; 5).

Количество размещений из n элементов по k обозначается Уравнения по комбинаторике с решениями(читается: «А из n по k», A — первая буква французского слова arrangement, что означает «размещение, приведение в порядок»). Как видим,Уравнения по комбинаторике с решениями

Выясним, сколько всего можно составить размещений из n элементов по k без повторений. Составление размещения представим себе как последовательное заполнение k мест, которые будем изображать в виде клеточек (рис. 21.1). На первое место можем выбрать один из n элементов данного множества (то есть элемент для первой клеточки можно выбрать n способами).

Если элементы нельзя повторять, то на второе место можно выбрать только один элемент из оставшихся, то есть из n – 1 элементов. Теперь уже два элемента использованы и на третье место можно выбрать только один из n – 2 элементов и т. д. На k-е место можно выбрать только один из n – (k –1) = n – k +1 элементов (см. рис. 21.1).

Уравнения по комбинаторике с решениями

Поскольку требуется выбрать элементы и на первое место, и на второе, . и на k-е, то используем правило произведения и получим следующую формулу числа размещений из n элементов по k:

Уравнения по комбинаторике с решениями

Например, Уравнения по комбинаторике с решениями(что совпадает с соответствующим значением, полученным выше). Аналогично можно обосновать формулу для нахождения числа размещений с повторениями. При решении простейших комбинаторных задач важно правильно выбрать формулу, по которой будут проводиться вычисления. Для этого нужно выяснить следующее:

  1. Учитывается ли порядок следования элементов в соединении?
  2. Все ли заданные элементы входят в полученное соединение?

Если, например, порядок следования элементов учитывается и из n данных элементов в соединении используется только k элементов, то по определению это — размещение из n элементов по k.

После определения вида соединения следует также выяснить, могут ли элементы в соединении повторяться, то есть выяснить, какую формулу необходимо использовать — для количества соединений без повторений или с повторениями.

Примеры решения задач:

Пример:

На соревнования по легкой атлетике приехала команда из 12 спортсменок. Сколькими способами тренер может определить, кто из них побежит в эстафете 4 × 100 м на первом, втором, третьем и четвертом этапах?

Решение:

Количество способов выбрать из 12 спортсменок четырех для участия в эстафете равно количеству размещений из 12 элементов по 4 (без повторений), то есть Уравнения по комбинаторике с решениями

Для выбора формулы выясняем ответы на вопросы, приведенные выше. Поскольку для спортсменок важно, в каком порядке они будут бежать, то порядок при выборе элементов учитывается. В полученное соединение входят не все 12 заданных элементов. Следовательно, соответствующее соединение — размещение из 12 элементов по 4 (без повторений, поскольку каждая спортсменка может бежать только на одном этапе эстафеты).

Пример:

Найдите количество трехзначных чисел, которые можно составить из цифр 1, 2, 3, 4, 5, 6, 7, если цифры в числе не повторяются.

Решение:

Количество трехзначных чисел, которые можно составить из семи цифр 1, 2, 3, 4, 5, 6, 7, равно числу размещений из 7 элементов по 3, то естьУравнения по комбинаторике с решениями

Для выбора формулы выясняем, что для чисел, которые мы будем составлять, порядок следования цифр учитывается и не все элементы выбираются (только 3 из заданных семи). Следовательно, соответствующее соединение — размещение из 7 элементов по 3 (без повторений).

Пример:

Найдите количество трехзначных чисел, которые можно составить из цифр 1, 2, 3, 4, 5, 6, 0, если цифры в числе не повторяются.

Выбор формулы проводится таким же образом, как и в задаче 2. Следует учесть, что если число, составленное из трех цифр, начинается цифрой 0, то оно не считается трехзначным. Следовательно, для ответа на вопрос задачи можно сначала из заданных 7 цифр записать все числа, состоящие из 3 цифр (см. задачу 2). Затем из количества полученных чисел вычесть количество чисел, составленных из трех цифр, но начинающихся цифрой 0. В последнем случае мы фактически будем из всех цифр без нуля (их 6) составлять двузначные числа. Тогда их количество равно числу размещений из 6 элементов по 2 (см. решение).

Можно выполнить также непосредственное вычисление, последовательно заполняя три места в трехзначном числе и используя правило произведения. В этом случае для наглядности удобно изображать соответствующие разряды в трехзначном числе в виде клеточек, например так:

Уравнения по комбинаторике с решениями

Решение:

Количество трехзначных чисел, которые можно составить из семи цифр (среди которых нет цифры 0), если цифры в числе не повторяются, равно числу размещений из 7 элементов по 3, то есть Уравнения по комбинаторике с решениями

Но среди данных цифр есть цифра 0, с которой не может начинаться трехзначное число. Поэтому из размещений из 7 элементов по 3 необходимо исключить те размещения, в которых первым элементом является цифра 0. Их количество равно числу размещений из 6 элементов по 2, то есть Уравнения по комбинаторике с решениямиСледовательно, искомое количество трехзначных чисел равно Уравнения по комбинаторике с решениями

Пример:

Решите уравнениеУравнения по комбинаторике с решениями

Решение:

ОДЗ: x ∈ N, Уравнения по комбинаторике с решениями. Тогда получаем: Уравнения по комбинаторике с решениями

На ОДЗ это уравнение равносильно уравнениям:

Тогда x = 0 или x = 5. В ОДЗ входит только x = 5.

Уравнения, в запись которых входят выражения, обозначающие количество соответствующих соединений из x элементов, считаются определенными только при натуральных значениях переменной x. Чтобы выражение Уравнения по комбинаторике с решениямиимело смысл, следует выбирать натуральные значения Уравнения по комбинаторике с решениями(в этом случае Уравнения по комбинаторике с решениямитакже существует и, конечно, Ax 2 ≠ 0). Для преобразования уравнения используем формулы:Уравнения по комбинаторике с решениями

Объяснение и обоснование:

Перестановкой из n элементов называется любое упорядоченное множество из n заданных элементов.

Напомним, что упорядоченное множество — это такое множество, для которого указано, какой элемент находится на первом месте, какой на втором, . какой на n-м.

Например, переставляя цифры в числе 236 (в котором множество цифр уже упорядоченное), можно составить такие перестановки без повторений: (2; 3; 6), (2; 6; 3), (3; 2; 6), (3; 6; 2), (6; 2; 3), (6; 3; 2) — всего 6 перестановок* .

Количество перестановок без повторений из n элементов обозначается Уравнения по комбинаторике с решениями(P — первая буква французского слова permutation — перестановка). Как видим, Уравнения по комбинаторике с решениями= 6.

Фактически перестановки без повторений из n элементов являются размещениями из n элементов по n без повторений, поэтому Уравнения по комбинаторике с решениямиПроизведение Уравнения по комбинаторике с решениямиобозначается n!. Поэтому полученная формула числа перестановок без повторений из n элементов может быть записана следующим образом:

Уравнения по комбинаторике с решениями

*Отметим, что каждая из перестановок определяет трехзначное число, составленное из цифр 2, 3, 6 таким образом, что цифры в числе не повторяются.

Например, Уравнения по комбинаторике с решениями(что совпадает с соответствующим значением, полученным выше).

С помощью факториалов формулу для числа размещений без повторений

Уравнения по комбинаторике с решениями(1)

запишем в другом виде. Для этого умножим и разделим выражение в формуле (1) на произведение Уравнения по комбинаторике с решениямитогда

Уравнения по комбинаторике с решениями

Следовательно, формула числа размещений без повторений из n элементов по k может быть записана так:

Уравнения по комбинаторике с решениями(2)

Для того чтобы этой формулой можно было пользоваться при всех значениях k, в частности при k = n – 1 и k = n, договорились считать, что

Например, по формуле (2) Уравнения по комбинаторике с решениями

Обратим внимание, что в тех случаях, когда значение n! оказывается очень большим, ответы оставляют записанными с помощью факториалов. Например,Уравнения по комбинаторике с решениями

Примеры решения задач:

Для выбора формулы при решении простейших комбинаторных задач достаточно выяснить следующее:

  1. Учитывается ли порядок следования элементов в соединении?
  2. Все ли заданные элементы входят в полученное соединение?

Если, например, порядок следования элементов учитывается и все n заданных элементов используются в соединении, то по определению это перестановки из n элементов.

Пример:

Найдите, сколькими способами можно восемь учащихся построить в колонну по одному.

Решение:

Количество способов равно числу перестановок из 8 элементов, то есть Уравнения по комбинаторике с решениями

Для выбора соответствующей формулы выясняем ответы на вопросы, приведенные выше. Поскольку порядок следования элементов учитывается и все 8 заданных элементов выбираются, то искомые соединения — это перестановки из 8 элементов без повторений. Их количество можно вычислить по формуле Уравнения по комбинаторике с решениями

Пример:

Найдите количество различных четырехзначных чисел, которые можно составить из цифр 0, 3, 7, 9 (цифры в числе не повторяются).

Решение:

Из четырех цифр 0, 3, 7, 9, не повторяя заданные цифры, можно получить Уравнения по комбинаторике с решениямиперестановок. Перестановки, начинающиеся с цифры 0, не являются записью четырехзначного числа — их количество Уравнения по комбинаторике с решениями. Тогда искомое количество четырехзначных чисел равноУравнения по комбинаторике с решениями

Поскольку порядок следования элементов учитывается и для получения четырехзначного числа надо использовать все элементы, то искомые соединения — это перестановки из 4 элементов. Их количество — Уравнения по комбинаторике с решениями. При этом необходимо учесть, что в четырехзначном числе на первом месте не может стоять цифра 0. Таких чисел будет столько, сколько раз мы сможем выполнить перестановки из 3 оставшихся цифр, то есть Уравнения по комбинаторике с решениями

Пример:

Имеется десять книг, из которых четыре — учебники. Сколькими способами можно поставить эти книги на полку так, чтобы все учебники стояли рядом?

Решение:

Сначала будем рассматривать учебники как одну книгу. Тогда на полке надо расставить не 10, а 7 книг. Это можно сделать Уравнения по комбинаторике с решениямиспособами. В каждом из полученных наборов книг можно выполнить еще Уравнения по комбинаторике с решениямиперестановок учебников. По правилу умножения искомое количество способов равноУравнения по комбинаторике с решениями

Задачу можно решать в два этапа. На первом будем условно считать все учебники одной книгой.

Тогда получим 7 книг (6 не учебников + 1 условная книга — учебник). Порядок следования элементов учитывается и используются все элементы (поставить на полку необходимо все книги). Следовательно, соответствующие соединения — это перестановки из 7 элементов. Их количество — Уравнения по комбинаторике с решениями.

На втором этапе решения будем переставлять между собой только учебники. Это можно сделать Уравнения по комбинаторике с решениямиспособами. Поскольку нам надо переставить и учебники, и другие книги, то используем правило произведения.

Объяснение и обоснование:

1. Сочетания без повторений:

Сочетанием без повторений из n элементов по k называется любое k-элементное подмножество заданного n-элементного множества.

Например, из множества можно составить следующие сочетания без повторений из трех элементов: , , , .

Количество сочетаний без повторений из n элементов по k элементов обозначается символом Уравнения по комбинаторике с решениями(читается: «число сочетаний из п по k» или «це из п по k», С — первая буква французского слова combinaison — сочетание). Как видим, Уравнения по комбинаторике с решениями

Выясним, сколько всего можно составить сочетаний без повторений из n элементов по k. Для этого используем известные нам формулы числа размещений и перестановок. Составление размещения без повторений из n элементов по k проведем в два этапа. Сначала выберем k разных элементов из заданного n-элементного множества, не учитывая порядок выбора этих элементов (то есть выберем kэлементное подмножество из n-элементного множества — сочетание без повторений из n-элементов по k). По нашему обозначению это можно сделать Уравнения по комбинаторике с решениямиспособами. После этого полученное множество из k разных элементов упорядочим. Его можно упорядочить Уравнения по комбинаторике с решениямиспособами. Получим размещения без повторений из n элементов по k. Следовательно, количество размещений без повторений из n элементов по k в k! раз больше числа сочетаний без повторений из n элементов по k, то естьУравнения по комбинаторике с решениямиОтсюда Уравнения по комбинаторике с решениямиУчитывая, что по формуле (2) Уравнения по комбинаторике с решениями, получаем:

Уравнения по комбинаторике с решениями(3)

Например, Уравнения по комбинаторике с решениямичто совпадает со значением, полученным выше.

Используя формулу (3), можно легко обосновать свойство 1 числа сочетаний без повторений, приведенное в табл. 28.

1) Поскольку Уравнения по комбинаторике с решениямито

Уравнения по комбинаторике с решениями(4)

Для того чтобы формулу (4) можно было использовать и при k = n, договорились считать, что Уравнения по комбинаторике с решениямиТогдаУравнения по комбинаторике с решениями

Заметим, что формулу (4) можно получить без вычислений с помощью достаточно простых комбинаторных рассуждений.

Когда мы выбираем k предметов из n, то n – k предметов мы оставляем. Если же, напротив, выбранные предметы оставим, а другие n – k -выберем, то получим способ выбора n – k предметов из n. Мы получили взаимно-однозначное соответствие способов выбора k и n – k предметов из n. Значит, количество одних и других способов одинаково. Но количество одних — Уравнения по комбинаторике с решениями, а других Уравнения по комбинаторике с решениями, поэтому Уравнения по комбинаторике с решениями.

Если в формуле (3) сократить числитель и знаменатель на (n – k)!, то получим формулу, по которой удобно вычислять Уравнения по комбинаторике с решениямипри малых значениях k:

Уравнения по комбинаторике с решениями(5)

Например,Уравнения по комбинаторике с решениями

2. Вычисление числа сочетаний без повторений с помощью треугольника Паскаля:

Для вычисления числа сочетаний без повторений можно применять формулу (3): Уравнения по комбинаторике с решениями, а можно последовательно вычислять соответствующие значения, пользуясь следующим свойством:

Уравнения по комбинаторике с решениями(6)

Для обоснования равенства (6) можно записать суммуУравнения по комбинаторике с решениями, используя формулу (3), и после приведения полученных дробей к общему знаменателю получить формулу для правой части равенства (6) (проделайте это самостоятельно). Также формулу (6) можно получить без вычислений с помощью комбинаторных рассуждений.

Уравнения по комбинаторике с решениями— это количество способов выбрать k +1 предмет из n + 1. Подсчитаем это количество, зафиксировав один предмет (назовем его «фиксированным»). Если мы не берем фиксированный предмет, то нам нужно выбрать k +1 предмет из n тех, что остались, а если мы его берем, то нужно выбрать из n тех, что остались, еще k предметов. Первое можно сделать Уравнения по комбинаторике с решениямиспособами, второеУравнения по комбинаторике с решениямиспособами. Всего как раз Уравнения по комбинаторике с решениямиспособов, следовательно,

Уравнения по комбинаторике с решениями

Это равенство позволяет последовательно вычислять значения Уравнения по комбинаторике с решениямис помощью специальной таблицы, которая называется треугольником Паскаля. Если считать, что Уравнения по комбинаторике с решениями, то он будет иметь вид, представленный в табл. 29.

Уравнения по комбинаторике с решениями

Каждая строка этой таблицы начинается с единицы и заканчивается единицейУравнения по комбинаторике с решениями

Если какая-либо строка уже заполнена, например третья, то в четвертой строке надо записать на первом месте единицу. На втором месте запишем число, равное сумме двух чисел третьей строки, стоящих над ним левее и правее (поскольку по формуле (6) Уравнения по комбинаторике с решениямиНа третьем месте запишем число, равное сумме двух следующих чисел третьей строки, стоящих над ним левее и правее Уравнения по комбинаторике с решениями, и т. д. (а на последнем месте снова запишем единицу).

Примеры решения задач:

Обратим внимание, что, как и раньше, для выбора формулы при решении простейших комбинаторных задач достаточно ответить на вопросы:

  1. Учитывается ли порядок следования элементов в соединении?
  2. Все ли заданные элементы входят в полученное соединение?

Чтобы выяснить, является ли заданное соединение сочетанием, достаточно ответить только на первый вопрос (см. схему в табл. 28). Если порядок следования элементов не учитывается, то по определению это сочетание из n элементов по k элементов.

Пример:

Из 12 членов туристической группы надо выбрать трех дежурных. Сколькими способами можно сделать этот выбор?

Решение:

Количество способов выбрать из 12 туристов трех дежурных равно количеству сочетаний из 12 элементов по 3 (без повторений), то естьУравнения по комбинаторике с решениями

Для выбора соответствующей формулы выясняем ответы на вопросы, приведенные выше. Поскольку порядок следования элементов не учитывается (для дежурных неважно, в каком порядке их выберут), то соответствующее соединение является сочетанием из 12 элементов по 3 (без повторений). Для вычисления можно использовать формулы (3) или (5), в данном случае применяем формулу (3):Уравнения по комбинаторике с решениями

Пример:

Из вазы с фруктами, в которой лежат 10 разных яблок и 5 разных груш, требуется выбрать 2 яблока и 3 груши. Сколькими способами можно сделать такой выбор?

Решение:

Выбрать 2 яблока из 10 можно Уравнения по комбинаторике с решениямиспособами. При каждом выборе яблок груши можно выбрать Уравнения по комбинаторике с решениямиспособами. Тогда по правилу произведения выбор требуемых фруктов можно выполнить Уравнения по комбинаторике с решениямиспособами. ПолучаемУравнения по комбинаторике с решениями

Сначала отдельно выберем 2 яблока из 10 и 3 груши из 5.

Поскольку при выборе яблок или груш порядок следования элементов не учитывается, то соответствующие соединения — сочетания без повторений.

Учитывая, что требуется выбрать 2 яблока и 3 груши, используем правило произведения и перемножим полученные возможности выбора яблок Уравнения по комбинаторике с решениямии груш Уравнения по комбинаторике с решениями

Бином Ньютона:

Уравнения по комбинаторике с решениями

Поскольку Уравнения по комбинаторике с решениями(при x ≠ 0 и a ≠ 0), то формулу бинома Ньютона можно записать еще и так:

Уравнения по комбинаторике с решениями

Общий член разложения степени бинома имеет вид

Уравнения по комбинаторике с решениями(где Уравнения по комбинаторике с решениями). Коэффициенты Уравнения по комбинаторике с решенияминазывают биномиальными коэффициентaми.

Свойства биномиальных коэффициентов:

  1. Число биномиальных коэффициентов (а следовательно, и число слагаемых) в разложении n-й степени бинома равно n + 1.
  2. Коэффициенты членов, равноудаленных от начала и конца разложения, равны между собой (поскольку Уравнения по комбинаторике с решениями)
  3. Сумма всех биномиальных коэффициентов равна Уравнения по комбинаторике с решениямиУравнения по комбинаторике с решениями
  4. Сумма биномиальных коэффициентов, стоящих на четных местах, равна сумме биномиальных коэффициентов, стоящих на нечетных местах.
  5. Для вычисления биномиальных коэффициентов можно воспользоваться треугольником Паскаля, в котором вычисления коэффициентов основываются на формуле Уравнения по комбинаторике с решениями

Уравнения по комбинаторике с решениями

Объяснение и обоснование:

Бином Ньютона:

Двучлен вида a + x также называют биномом. Из курса алгебры известно, что:

Уравнения по комбинаторике с решениями

Можно заметить, что коэффициенты разложения степени бинома Уравнения по комбинаторике с решениямипри n = 1, 2, 3 совпадают с числами в соответствующей строке треугольника Паскаля. Оказывается, что это свойство выполняется для любого натурального n, то есть справедлива формула

Уравнения по комбинаторике с решениями(7)

Формулу (7) называют биномом Ньютона. Правая часть этого равенства называется разложением степени биномаУравнения по комбинаторике с решениями, а числа Уравнения по комбинаторике с решениями(при k = 0, 1, 2, . n) называют биномиальными коэффициентами.

Общий член разложения степени бинома имеет вид

Уравнения по комбинаторике с решениями

Обосновать формулу (7) можно, например, с помощью метода математической индукции. (Проведите такое обоснование самостоятельно.)

Приведем также комбинаторные рассуждения для обоснования формулы бинома Ньютона.

По определению степени с натуральным показателем Уравнения по комбинаторике с решениями Уравнения по комбинаторике с решениями(всего n скобок). Раскрывая скобки, получаем в каждом слагаемом произведение n букв, каждая из которых — а или х. Если, например, в каком-либо слагаемом количество букв x равно k, то количество букв а в нем — n – k, то есть каждое слагаемое имеет вид Уравнения по комбинаторике с решениямипри некотором k от 0 до n. Покажем, что для каждого такого k число слагаемых anУравнения по комбинаторике с решениямиравно Уравнения по комбинаторике с решениями, откуда после приведения подобных членов и получаем формулу бинома. Произведение Уравнения по комбинаторике с решениямиполучаем, взяв букву x из k скобок и букву а из n – k тех скобок, которые остались. Разные такие слагаемые получим путем разного выбора первых k скобок, а k скобок из n можно выбрать именно Уравнения по комбинаторике с решениямиспособами. Следовательно, общий член разложения бинома Уравнения по комбинаторике с решениямидействительно имеет вид Уравнения по комбинаторике с решениямигде k = 0, 1, 2, . n.

Именно из-за бинома Ньютона числа Уравнения по комбинаторике с решениямичасто называют биномиальными коэффициентами.

Записывая степень двучлена по формуле бинома Ньютона для небольших значений n, биномиальные коэффициенты можно вычислять с помощью треугольника Паскаля (см. табл. 30).

Например, Уравнения по комбинаторике с решениями

Так как Уравнения по комбинаторике с решениями, формулу бинома Ньютона можно записать в виде:

Уравнения по комбинаторике с решениями(8)

Если в формуле бинома Ньютона (8) заменить x на (–x), то получим формулу возведения в степень разности a – x:

Уравнения по комбинаторике с решениями

Например, Уравнения по комбинаторике с решениями(знаки членов разложения чередуются!).

Свойства биномиальных коэффициентов:

  1. Число биномиальных коэффициентов (а следовательно, и число слагаемых) в разложении n-й степени бинома равно n + 1, поскольку разложение содержит все степени x от 0 до n (и других слагаемых не содержит).
  2. Коэффициенты членов, равноудаленных от начала и конца разложения, равны между собой, поскольку Уравнения по комбинаторике с решениями
  3. Сумма всех биномиальных коэффициентов равнаУравнения по комбинаторике с решениями

Для обоснования полагаем в равенстве (7) значения a = x = 1 и получаем:

Уравнения по комбинаторике с решениями

Например, Уравнения по комбинаторике с решениями

4. Сумма биномиальных коэффициентов, стоящих на четных местах, равна сумме биномиальных коэффициентов, стоящих на нечетных местах.

Для обоснования возьмем в равенстве (7) значения a = 1, x = –1:

Уравнения по комбинаторике с решениями

Тогда Уравнения по комбинаторике с решениями

Примеры решения задач:

Пример:

По формуле бинома Ньютона найдите разложение степениУравнения по комбинаторике с решениями.

Для нахождения коэффициентов разложения можно использовать треугольник Паскаля (табл. 30) или вычислять их по общей формуле. По треугольнику Паскаля коэффициенты равны: 1, 6, 15, 20, 15, 6, 1. Учитывая, что при возведении разности в степень знаки членов разложения чередуются, получаем:

Уравнения по комбинаторике с решениями

Для упрощения записи ответа можно избавиться от иррациональности в знаменателях полученных выражений (см. решение) или сначала учесть, что ОДЗ данного выражения: x > 0. Тогда Уравнения по комбинаторике с решениямито есть данное выражение можно записать так: Уравнения по комбинаторике с решениямии возвести в степень последнее выражение.

Решение:

Уравнения по комбинаторике с решениями

Пример:

В разложении степени Уравнения по комбинаторике с решенияминайдите член, содержащий Уравнения по комбинаторике с решениями

Решение:

Уравнения по комбинаторике с решениями.

Общий член разложения: Уравнения по комбинаторике с решениями

По условию член разложения должен содержать Уравнения по комбинаторике с решениями, следовательно, Уравнения по комбинаторике с решениямиОтсюда k = 6.

Тогда член разложения, содержащий Уравнения по комбинаторике с решениями, равен

Уравнения по комбинаторике с решениями

На ОДЗ (b > 0) каждое слагаемое в данном двучлене можно записать как степень с дробным показателем. Это позволит проще записать общий член разложения степени Уравнения по комбинаторике с решениями

Уравнения по комбинаторике с решениями

(где k = 0, 1, 2, . n), выяснить, какой из членов разложения содержит Уравнения по комбинаторике с решениямии записать его. Чтобы упростить запись общего члена разложения, запишем:

Уравнения по комбинаторике с решениями

Всё о комбинаторике

Пусть имеется несколько множеств элементов:

Уравнения по комбинаторике с решениями

Вопрос: сколькими способами можно составить новое множество Уравнения по комбинаторике с решениямивзяв из каждого исходного множества по одному элементу? Ответ на этот вопрос дают следующие рассуждения.

Элемент Уравнения по комбинаторике с решениямииз первого множества можно выбрать Уравнения по комбинаторике с решениямиспособами, элемент Уравнения по комбинаторике с решениямииз второго – s способами, элемент с можно выбрать Уравнения по комбинаторике с решениямиспособами и т. д. Пару элементов Уравнения по комбинаторике с решениямиможно составить Уравнения по комбинаторике с решениямиs способами. Это следует из табл. 1.1, в которой перечислены все способы такого выбора.

Уравнения по комбинаторике с решениями

Способы выбора трех элементов аbc перечислены в табл. 1.2.

Уравнения по комбинаторике с решениями

В этой таблице Уравнения по комбинаторике с решениямистрок и Уравнения по комбинаторике с решениямиs столбцов. Поэтому искомое число способов выбора трех элементов аbc равно Уравнения по комбинаторике с решениямиs Уравнения по комбинаторике с решениями. Продолжая рассуждать подобным образом, получим следующее утверждение.

Основной комбинаторный принцип. Если некоторый первый выбор можно сделать Уравнения по комбинаторике с решениями способами, для каждого первого выбора некоторый второй можно сделать s способами, для каждой пары первых двух – третий выбор можно сделать Уравнения по комбинаторике с решениями способами и т.д., то число способов для последовательности таких выборов равно Уравнения по комбинаторике с решениямиs Уравнения по комбинаторике с решениями.

Комбинаторные формулы в прикладных задачах теории вероятностей обычно связывают с выбором Уравнения по комбинаторике с решениямиэлементов («выборкой объема Уравнения по комбинаторике с решениями») из совокупности, состоящей из Уравнения по комбинаторике с решениямиэлементов (элементов «генеральной совокупности»). Различают два способа выбора:

  • а) повторный выбор, при котором выбранный элемент возвращается в генеральную совокупность и может быть выбран вновь;
  • б) бесповторный выбор, при котором выбранный элемент в совокупность не возвращается и выборка не содержит повторяющихся элементов.

При повторном выборе каждый по порядку элемент может быть выбран Уравнения по комбинаторике с решениямиспособами. Согласно комбинаторному принципу, такую выборку можно сделать Уравнения по комбинаторике с решениямиспособами. Например, повторную выборку объема 2 из трех элементов Уравнения по комбинаторике с решениямиможно сделать 3 2 =9 способами: Уравнения по комбинаторике с решениямиУравнения по комбинаторике с решениями

В случае бесповторной выборки первый элемент можно выбрать Уравнения по комбинаторике с решениямиспособами, для второго остается Уравнения по комбинаторике с решениямивозможность выбора, третий элемент можно выбрать Уравнения по комбинаторике с решениямиспособами и т.д. Элемент выборки с номером Уравнения по комбинаторике с решениямиможно выбрать Уравнения по комбинаторике с решениямиспособом. Согласно комбинаторному принципу, общее число бесповторных выборок объема Уравнения по комбинаторике с решениямиравно

Уравнения по комбинаторике с решениями

Число Уравнения по комбинаторике с решенияминазывают числом размещений из Уравнения по комбинаторике с решениямиэлементов по Уравнения по комбинаторике с решениями.

Например, существует Уравнения по комбинаторике с решениямиразмещений из трех элементов Уравнения по комбинаторике с решениямипо два: Уравнения по комбинаторике с решениямиОтметим, что и в первом случае и во втором выборки отличаются либо составом элементов, либо порядком выбора элементов.

Выделим особо случай, когда один за другим выбраны все Уравнения по комбинаторике с решениямиэлементов. В этом случае выборки имеют один и тот же состав (все Уравнения по комбинаторике с решениямиэлементов) и отличаются только порядком выбора элементов. Поэтому число

Уравнения по комбинаторике с решениями

называют числом перестановок из Уравнения по комбинаторике с решениямиэлементов.

Например, пять человек могут встать в очередь Уравнения по комбинаторике с решениямиспособами. Три элемента Уравнения по комбинаторике с решениямиможно переставить Уравнения по комбинаторике с решениямиспособами: Уравнения по комбинаторике с решениями

Подсчитаем количество бесповторных выборок объема Уравнения по комбинаторике с решениями, которые отличаются друг от друга только составом элементов. Пусть X — число таких выборок. Для каждого набора из Уравнения по комбинаторике с решениямиэлементов можно выбрать порядок их расположения Уравнения по комбинаторике с решениямиспособами. Тогда Уравнения по комбинаторике с решениямиравно числу способов выбрать Уравнения по комбинаторике с решениямиразличных элементов и выбрать порядок их расположения, т.е. равно числу размещений из Уравнения по комбинаторике с решениямиэлементов по Уравнения по комбинаторике с решениями:

Уравнения по комбинаторике с решениями

Это число называют числом сочетаний из Уравнения по комбинаторике с решениямиэлементов по Уравнения по комбинаторике с решениями и обозначают через Уравнения по комбинаторике с решениямиЕсли в формуле (1.2) умножить числитель и знаменатель на Уравнения по комбинаторике с решениями, то

Уравнения по комбинаторике с решениями

Например, сочетаний из четырех элементов Уравнения по комбинаторике с решениямипо два существует Уравнения по комбинаторике с решениями. Это Уравнения по комбинаторике с решениями

Так как из Уравнения по комбинаторике с решениями элементов выбрать Уравнения по комбинаторике с решениями элементов можно единственным образом, то Уравнения по комбинаторике с решениямиоткуда следует, что Уравнения по комбинаторике с решениями

Величины Уравнения по комбинаторике с решенияминазывают биномиальными коэффициентами. Название связано с формулой бинома Ньютона

Уравнения по комбинаторике с решениями

Из формулы (1.3) следует, что

Уравнения по комбинаторике с решениями

Биномиальные коэффициенты образуют так называемый треугольник Паскаля, который имеет вид:

Уравнения по комбинаторике с решениями

В Уравнения по комбинаторике с решениями-й строке треугольника Паскаля располагаются коэффициенты, соответствующие представлению Уравнения по комбинаторике с решениямипо формуле (1.3). Треугольником удобно пользоваться для нахождения значений Уравнения по комбинаторике с решениями. Это значение находится на пересечении Уравнения по комбинаторике с решениями-й строки и Уравнения по комбинаторике с решениями-го наклонного ряда. Например, Уравнения по комбинаторике с решениями

Биномиальные коэффициенты обладают свойством симметрии:

Уравнения по комбинаторике с решениями

Это наглядно демонстрирует треугольник Паскаля. Равенство (1.4) подтверждает тот очевидный факт, что выбор Уравнения по комбинаторике с решениями элементов из n равносилен выбору тех Уравнения по комбинаторике с решениямиУравнения по комбинаторике с решениями элементов из Уравнения по комбинаторике с решениями, которые следует удалить, чтобы остались Уравнения по комбинаторике с решениями элементов.

При повторном выборе из Уравнения по комбинаторике с решениями элементов число выборок объема Уравнения по комбинаторике с решениями, которые отличаются только составом равно Уравнения по комбинаторике с решениямиЕще раз подчеркнем, что речь идет о выборках, которые отличаются хотя бы одним элементом, а порядок выбора этих элементов во внимание не принимается. Число таких выборок можно подсчитать следующим образом. Между элементами Уравнения по комбинаторике с решениямипоставим разграничительные знаки, например, нули: Уравнения по комбинаторике с решениямиТаких знаков (нулей) понадобится Уравнения по комбинаторике с решениями. На месте каждого элемента поставим столько единиц, сколько раз предполагается выбрать этот элемент. Например, комбинация Уравнения по комбинаторике с решениямиозначает, что элемент Уравнения по комбинаторике с решениямивыбран четыре раза, элемент Уравнения по комбинаторике с решениямивыбран один раз, элемент Уравнения по комбинаторике с решениямине выбран, . элемент Уравнения по комбинаторике с решениямивыбран два раза. Заметим, что в такой записи число единиц равно объему выборки Уравнения по комбинаторике с решениями. Для перебора всех возможных комбинаций нужно из Уравнения по комбинаторике с решениямимест выбрать Уравнения по комбинаторике с решениямиместо и поставить на них нули, а на остальных местах разместить единицы. Это можно сделать способами.

Уравнения по комбинаторике с решениями

Совокупность из Уравнения по комбинаторике с решениями элементов разделить на Уравнения по комбинаторике с решениямигрупп по Уравнения по комбинаторике с решениямиэлементов соответственно Уравнения по комбинаторике с решениямиможно Уравнения по комбинаторике с решениямиспособами. Порядок элементов внутри каждой из этих Уравнения по комбинаторике с решениямигрупп не имеет значения.

Пусть Уравнения по комбинаторике с решениями– множества, число элементов в каждом из которых равно соответственно Уравнения по комбинаторике с решениямиСоставить множество B из Уравнения по комбинаторике с решениямиэлементов множества А1, Уравнения по комбинаторике с решениямиэлементов множества А2, …, Уравнения по комбинаторике с решениямиэлементов множества Аk, можно, согласно основному комбинаторному принципу, способами.

Уравнения по комбинаторике с решениями

Для безошибочного выбора комбинаторной формулы достаточно последовательно ответить на вопросы в следующей схеме:

Уравнения по комбинаторике с решениями

Например, число словарей, необходимых для непосредственного перевода с одного на другой, для пяти языков определяется из следующих рассуждений. Для составления словаря выбираем из пяти языков (Уравнения по комбинаторике с решениями= 5) любые два (Уравнения по комбинаторике с решениями=2). Выбор бесповторный, причем при выборе важен и состав выбора и порядок выбора. Поэтому искомое число словарей равно Уравнения по комбинаторике с решениями

Комбинаторные задачи с решением

Комбинаторика — раздел математики, занимающийся вопросом выбора и расположения элементов некоторого конечного множества в соответствии с заданными условиями.

Рассмотрим примеры задач комбинаторики.

Пример №1

Сколькими способами можно выбрать путь из начала координат 0(0,0) в точку В(6,4), если каждый шаг равен единице, но его можно совершать только вправо или вверх? Сколько таких путей проходит через точку А(2,3)?

Решение. Весь путь занимает 10 шагов (четыре вверх и шесть вправо). Для планирования пути следует решить, какие именно по счету четыре шага следует сделать вверх, а остальные шесть — вправо. Выбор бесповторный и нас интересует только состав выбора. Поэтому в описанных условиях всего путей из точки О в точку В будет Уравнения по комбинаторике с решениями

Рассуждая подобным образом легко видеть, что путей из точки О в точку А существует Уравнения по комбинаторике с решениямиа путь из точки А в точку В можно выбрать Уравнения по комбинаторике с решениямиспособами. По комбинаторному принципу всего путей через точку А существует 10 • 5 = 50.

Пример №2

Сколькими способами можно выбрать путь из начала координат 0(0,0) в точку Уравнения по комбинаторике с решениямиесли каждый шаг равен 1, но его можно совершать только вправо или вверх? Сколько таких путей проходит через точку Уравнения по комбинаторике с решениями(См. пример 1.1 и исходные данные.)

Исходные данные к задаче 1.1.

Уравнения по комбинаторике с решениями

Пример №3

В городе с идеальной прямоугольной планировкой (сеть улиц в этом городе изображена на рис. 1.1) из пункта А выходят Уравнения по комбинаторике с решениямичеловек. Половина из них идет по направлению Уравнения по комбинаторике с решениямиполовина — по направлению Уравнения по комбинаторике с решениямиДойдя до первого перекрестка, каждая группа разделяется так, что половина ее идет по направлению Уравнения по комбинаторике с решениямиполовина — по направлению Уравнения по комбинаторике с решениямиТакое же разделение происходит на каждом перекрестке. Требуется перечислить перекрестки, на которых окажутся люди после прохождения N улиц (отрезков на рис. 1.1), и сколько людей окажется на каждом из этих перекрестков.

Уравнения по комбинаторике с решениями

Решение. Каждый человек пройдет N улиц и окажется на одном из перекрестков Уравнения по комбинаторике с решениямиКоординаты перекрестков указаны в предположении, что точка А служит началом координат.

На каждом перекрестке для каждого человека производится выбор из двух возможностей: идти в направлении Уравнения по комбинаторике с решениямиили в направлении Уравнения по комбинаторике с решениямиПоэтому всего возможных путей будет Уравнения по комбинаторике с решениями. Из этого следует, что каждый путь пройдет только один человек.

В пункте Уравнения по комбинаторике с решениямиокажется столько человек, сколько различных путей ведет в этот пункт из точки А . Чтобы попасть в пункт Уравнения по комбинаторике с решенияминеобходимо из N улиц выбрать бесповторным способом к улиц в направлении Уравнения по комбинаторике с решениями. Это можно сделать Уравнения по комбинаторике с решениямиспособами.

Ответ. Уравнения по комбинаторике с решениями

Пример №4

Сколькими способами можно Уравнения по комбинаторике с решениями одинаковых предметов распределить между Уравнения по комбинаторике с решениямилицами так, чтобы каждый получил не менее одного предмета?

Решение. Поставим эти предметы в ряд. Между ними будет Уравнения по комбинаторике с решениямипромежуток. В любые Уравнения по комбинаторике с решениямииз этих промежутков поставим разделяющие перегородки. Тогда все предметы разделятся на Уравнения по комбинаторике с решенияминепустых частей. Первую часть передадим первому лицу, вторую — второму и т.д. Выбрать же Уравнения по комбинаторике с решениямипромежуток из Уравнения по комбинаторике с решениямипромежутка можно Уравнения по комбинаторике с решениямиспособами. Заметим, что вообще Уравнения по комбинаторике с решениями предметов распределить между Уравнения по комбинаторике с решениямилицами можно Уравнения по комбинаторике с решениямиспособами.

Ответ. Уравнения по комбинаторике с решениями

Пример 1.4.

Сколькими способами можно распределить 6 яблок, 8 груш и 10 слив между тремя детьми? Сколькими способами это можно сделать так, чтобы каждый ребенок получил по меньшей мере одно яблоко, одну сливу и одну грушу?

Решение. Яблоки в соответствии с формулой (1.5) можно распределить Уравнения по комбинаторике с решениямиспособами, груши — Уравнения по комбинаторике с решениями, а сливы Уравнения по комбинаторике с решениямиспособами. По комбинаторному принципу всего способов Уравнения по комбинаторике с решениямиЕсли необходимо, чтобы каждый ребенок получил по меньшей мере одно яблоко, одну грушу и одну сливу, то в соответствии с формулой предыдущего примера имеем Уравнения по комбинаторике с решениямиспособов.

Пример №5

Сколько цифр в первой тысяче не содержат в своей записи цифры 5?

Решение. Для записи любой из цифр 000, 001, 002, . 999 необходимо трижды выбрать повторным способом одну из десяти цифр, поэтому и получается всего Уравнения по комбинаторике с решениямичисел. Если цифру 5 исключить, то выбор можно производить только из девяти цифр: 0, 1,2, 3, 4, 6, 7, 8, 9. Поэтому всего получится Уравнения по комбинаторике с решениямичисел в первой тысяче, в записи которых нет цифры 5.

Пример №6

Сколько шестизначных чисел содержат в записи ровно три различных цифры?

Решение. Заметим, что всего шестизначных чисел имеется Уравнения по комбинаторике с решениями, так как первая цифра может быть любой (исключая нуль), а остальные пять могут быть выбраны Уравнения по комбинаторике с решениямиспособами.

Выбрать три ненулевых цифры можно Уравнения по комбинаторике с решениямиспособами. Из выбранных трех цифр можно составить Уравнения по комбинаторике с решениямишестизначных чисел, из двух — Уравнения по комбинаторике с решениями, а из одной — Уравнения по комбинаторике с решениямишестизначное число. По формуле (1.7) получаем, что существует Уравнения по комбинаторике с решениямишестизначных чисел, в записи которых есть только три заданные цифры. Поэтому общее число шестизначных чисел, в записи которых имеются три отличные от нуля цифры, равно Уравнения по комбинаторике с решениями

Учтем теперь возможность использования нуля. К нулю нужно добавить две цифры, что можно сделать Уравнения по комбинаторике с решениямиспособами. Если, например, были выбраны цифры 0, 2, 5, то первой цифрой должна быть 2 или 5. К этой первой цифре в соответствии с формулой (1.7) можно добавить Уравнения по комбинаторике с решениямикомбинаций остальных пяти цифр. Тогда всего шестизначных чисел, состоящих из 0, 2, 5 будет Уравнения по комбинаторике с решениямиВсего же шестизначных чисел, записанных тремя цифрами, среди которых встречается нуль, ровно Уравнения по комбинаторике с решениямиВсего чисел, удовлетворяющих условиям задачи, имеется Уравнения по комбинаторике с решениями

Пример №7

В саду есть цветы десяти наименований (розы, флоксы, ромашки и т. д.).

а) Сколькими способами можно составить букет из пяти цветков (не принимая во внимание совместимость растений и художественные соображения)?

б) Сколькими способами можно составить букет из пяти различных цветков?

в) Сколькими способами можно составить букет из пяти цветков так, чтобы в букете непременно было хотя бы по одному цветку двух определенных наименований

Решение. а) Если запрета на повторение цветков нет, то мы имеем дело с повторным выбором и нас интересует только состав. Поэтому по формуле (1.5) получаем Уравнения по комбинаторике с решениямиспособа.

б) Если цветы должны быть разными, то способ выбора бесповторный и букет можно составить Уравнения по комбинаторике с решениямиспособами.

в) Отберем по одному цветку каждого из двух названных наименований. Три остальных цветка можно выбрать из 10 возможных Уравнения по комбинаторике с решениямиспособами.

Ответ. а) 2002; б) 504; в) 220.

Пример №8

Имеется Уравнения по комбинаторике с решениямияблок, Уравнения по комбинаторике с решениямигруш и Уравнения по комбинаторике с решениямиперсиков. Сколькими способами можно их разложить по двум корзинам? Сколькими способами можно это сделать, если в каждой корзине должно быть хотя бы по одному фрукту всех названных видов (полагаем, что фруктов каждого наименования два или больше)?

Решение. Ясно, что яблоки можно разложить Уравнения по комбинаторике с решениямиспособом (в первую корзину можно не положить яблок совсем, положить одно яблоко, два яблока, …, все яблоки). Те же рассуждения в отношении груш и персиков дают соответственно Уравнения по комбинаторике с решениямикомбинаций. По комбинаторному принципу всего будет Уравнения по комбинаторике с решениямиспособов.

При ответе на второй вопрос учтем, что следует по одному яблоку сразу положить в каждую из корзин, а остальные Уравнения по комбинаторике с решениямияблока раскладывать произвольным образом (в первую корзину либо не добавляем яблок, либо добавляем одно, либо –– два, …, либо – все Уравнения по комбинаторике с решениямияблока). Все это можно сделать Уравнения по комбинаторике с решениямиспособами. Те же рассуждения насчет других фруктов и комбинаторный принцип дают следующий результат: Уравнения по комбинаторике с решениями

Ответ. Уравнения по комбинаторике с решениями

Пример №9

Требуется найти число натуральных делителей натурального числа Уравнения по комбинаторике с решениями.

Решение. Разложим Уравнения по комбинаторике с решениямина простые множители:

Уравнения по комбинаторике с решениями

где Уравнения по комбинаторике с решениями– различные простые числа. (Например, Уравнения по комбинаторике с решениямиУравнения по комбинаторике с решениями)

Заметим, что при разделении числа Уравнения по комбинаторике с решениямина любые два множителя Уравнения по комбинаторике с решениямии Уравнения по комбинаторике с решениямипростые сомножители распределятся между Уравнения по комбинаторике с решениямии Уравнения по комбинаторике с решениями. Если сомножитель , Уравнения по комбинаторике с решениямив число Уравнения по комбинаторике с решениямивходит Уравнения по комбинаторике с решениямито разложение (1.8) примет вид:

Уравнения по комбинаторике с решениями

Так что разложение Уравнения по комбинаторике с решениямина два сомножителя сводится к разделению каждого из чисел Уравнения по комбинаторике с решениямина две части, а это можно сделать Уравнения по комбинаторике с решениямиспособами.

Ответ. Уравнения по комбинаторике с решениями.

Пример №10

Сколькими способами легкоатлет, собираясь на тренировку, может выбрать себе пару спортивной обуви, имея 5 пар кроссовок и 2 нары кед?

Очевидно, что выбрать одну из имеющихся пар обуви, кроссовки или кеды, можно 5 + 2 = 7 способами.

Обобщая, приходим к комбинаторному правилу сложения:

  • если некоторый элемент Уравнения по комбинаторике с решениямиможно выбрать Уравнения по комбинаторике с решениямиспособами, а элемент Уравнения по комбинаторике с решениями(независимо от выбора элемента Уравнения по комбинаторике с решениями) — Уравнения по комбинаторике с решениямиспособами, то выбрать Уравнения по комбинаторике с решениямиилиУравнения по комбинаторике с решениямиможно Уравнения по комбинаторике с решениямиспособами.

Это правило справедливо также для трех и более элементов.

Пример №11

В меню школьной столовой предлагается на выбор 4 вида пирожков и 3 вида сока. Сколько разных вариантов выбора завтрака, состоящего из одного пирожка и одного стакана сока, имеется у учащегося этой школы? Уравнения по комбинаторике с решениями

Пирожок можно выбрать 4 способами и к каждому пирожку выбрать сок 3 способами (рис. 76). Следовательно, учащийся имеет Уравнения по комбинаторике с решениямивариантов выбора завтрака.

Обобщая, приходим к комбинаторному правилу умножения:

  • если некоторый элемент Уравнения по комбинаторике с решениямиможно выбрать Уравнения по комбинаторике с решениями, способами и после каждого такого выбора (независимо от выбора элемента Уравнения по комбинаторике с решениями) другой элемент Уравнения по комбинаторике с решениямиможно выбрать Уравнения по комбинаторике с решениямиспособами, то пару объектов Уравнения по комбинаторике с решениямииУравнения по комбинаторике с решениямиможно выбрать Уравнения по комбинаторике с решениямиспособами.

Это правило справедливо также для трех и более элементов.

Пример №12

Сколько трехзначных чисел можно составить из цифр 1, 2, 3, 4, если в числе: 1) цифры не повторяются; 2) цифры могут повторяться?

Уравнения по комбинаторике с решениями

Решение:

1) Первую цифру можем выбрать 4 способами (рис.77). Так как после выбора первой цифры их останется три (ведь цифры в нашем случае повторяться не могут), то вторую цифру можем выбрать 3 способами.И наконец, третью цифру можем выбрать из оставшихся двух — то есть 2 способами. Следовательно, количество искомых трехзначных у чисел будет равно Уравнения по комбинаторике с решениями.

2) Применим комбинаторное правило умножения. Так как цифры в числе могут повторяться, то каждую из цифр искомого числа можно выбрать 4 способами (рис. 78), и тогда таких чисел будет Уравнения по комбинаторике с решениями.

Ответ. 1) 24 числа; 2) 64 числа.

Отметим, что решить подобные задачи без применения комбинаторного правила умножения можно только путем перебора всех возможных вариантов чисел, удовлетворяющих условию задачи. Но такой способ решения является слишком долгим и громоздким.

Пример №13

Сколько четных пятизначных чисел можно составить из цифр 5, 6, 7, 8, 9, если цифры в числе не повторяются?

Решение:

Четное пятизначное число можно получить, если последней его цифрой будет 6 или 8. Чисел, у которых последней является цифра 6, будет Уравнения по комбинаторике с решениями(рис. 79),

Уравнения по комбинаторике с решениями

а тех, у которых последней является цифра 8, — также 24. По комбинаторному правилу сложения всего четных чисел будет Уравнения по комбинаторике с решениями.

Пример №14

Азбука племени АБАБ содержит всего две буквы — «а» и «б». Сколько слов в языке этого племени состоит: 1) из двух букв; 2) из трех букв?

Решение:

1) аа, ба, аб, бб (всего четыре слова); 2) ааа, ааб, аба, абб, ббб, бба, баб, баа (всего восемь слов).

Заметим, что найденное количество слов соответствует комбинаторному правилу умножения. Так как на каждое место есть два «претендента» — «а» и «б», то слов, состоящих из двух букв, будет Уравнения по комбинаторике с решениями, а из трех букв — Уравнения по комбинаторике с решениями.

Пример №15

В футбольной команде из 11 игроков надо выбрать капитана и его заместителя. Сколькими способами это можно сделать?

Решение:

Капитаном можно выбрать любого из 11 игроков, а его заместителем — любого из 10 оставшихся игроков. Таким образом (по правилу умножения), имеем Уравнения по комбинаторике с решениямиразных способов.

Пример №16

В Стране Чудес 10 городов и каждые два из них соединяет авиалиния. Сколько авиалиний в этой стране?

Решение. Так как каждая авиалиния соединяет два города, то одним из них может быть любой из 10 городов, а другим — любой из 9 оставшихся. Следовательно, количество авиалиний равно Уравнения по комбинаторике с решениями. Но при этом каждую из авиалиний мы учли дважды. Поэтому всего их будет Уравнения по комбинаторике с решениями.

Комбинаторные задачи неразрывно связаны с задачами теории вероятностей, еще одного раздела математики.

В ХIII-ХII в. до н. э. встречаются упоминания о вопросах, близких к комбинаторным. Некоторые комбинаторные задачи решали и в Древней Греции. В частности, Аристоксен из Тарента (IV в. до н. э.), ученик Аристотеля, перечислил различные комбинации длинных и коротких слогов в стихотворных размерах. А Папп Александрийский в IV в. н. э. рассматривал число пар и троек, которые можно получить из трех элементов, допуская их повторения. Некоторые элементы комбинаторики были известны и в Индии во II в. до н. э. Индийцы умели вычислять числа, известные нам как коэффициенты формулы бинома Ньютона. Позднее, в VIII в. н. э., арабы нашли и саму эту формулу, и ее коэффициенты, которые сейчас вычисляют с помощью комбинаторных формул или «треугольника Паскаля».

Свой нынешний вид упомянутые комбинаторные формулы приобрели благодаря средневековому ученому Леви бен Гершону (XIV в.) и французскому математику П. Эригону (XVII в.).

В III в. н. э. сирийский философ Порфирий для классификации понятий составил специальную схему, получившую название «древо Порфирия». Сейчас подобные деревья используются для решения определенных задач комбинаторики в разнообразных областях знаний. Некоторые ранее неизвестные комбинаторные задачи рассмотрел Леонардо Пизанский (Фибоначчи) в своей знаменитой «Книге абака» (1202 г.), в частности, о нахождении наименьшего набора различных гирь, позволяющего взвесить груз с любой целочисленной массой, не превышающей заданного числа. Со времен греческих математиков были известны две последовательности, каждый член которых получали по определенному правилу из предыдущих, — арифметическая и геометрическая прогрессии. А Фибоначчи впервые в одной из задач выразил член последовательности через два предыдущих, используя формулу, которую назвали рекуррентной. В дальнейшем метод рекуррентных формул стал одним из мощнейших для решения комбинаторных задач.

Как ни странно, развитию комбинаторики в значительной степени способствовали азартные игры, которые были очень популярны в XVI в. В частности, вопросами определения разнообразных комбинаций в игре в кости в то время занимались такие известные итальянские математики, как Д. Кардано, H. Тарталья и др. А наиболее полно изучил этот вопрос в XVII в. Галилео Галилей.

Современные комбинаторные задачи высокого уровня сложности связаны с объектами в других отраслях математики: определителями, конечными геометриями, группами, математической логикой и т. п.

Правила суммы и произведения

Вспомните, что в математике любые совокупности называют множествами. Объекты, входящие в множества, называют его элементами. Множества обозначают большими латинскими буквами, а их элементы записывают в фигурных скобках. Считают, что все элементы множества различны.

Например, Уравнения по комбинаторике с решениями

Множества бывают конечными и бесконечными. Если множество не содержит ни одного элемента, его называют пустым и обозначают символом Уравнения по комбинаторике с решениями

Два множества называют равными, если они состоят из одних и тех же элементов.

Если Уравнения по комбинаторике с решениями— часть множества Уравнения по комбинаторике с решениямито его называют подмножеством множества Уравнения по комбинаторике с решениямии записывают Уравнения по комбинаторике с решениямиНаглядно это изображают с помощью диаграммы Эйлера (рис. 135, а). В частности, для числовых множеств правильные такие соотношения:

Уравнения по комбинаторике с решениями

Случается, что множества Уравнения по комбинаторике с решениямиимеют общие элементы. Если множество Уравнения по комбинаторике с решениямисодержит все общие элементы множеств Уравнения по комбинаторике с решениямии только их, то множество Уравнения по комбинаторике с решенияминазывают пересечением множеств Уравнения по комбинаторике с решениямиЗаписывают это так: Уравнения по комбинаторике с решениямиДиаграммой Эйлера пересечение изображают, как показано на рисунке 135, б. Множество, содержащее каждый элемент каждого из множеств Уравнения по комбинаторике с решениямии только эти

Уравнения по комбинаторике с решениями

элементы, называется объединением множеств Уравнения по комбинаторике с решениямиЕсли Уравнения по комбинаторике с решениями— объединение множеств Уравнения по комбинаторике с решениямито пишут Уравнения по комбинаторике с решениями(рис. 135, в).

Разницей множеств Уравнения по комбинаторике с решенияминазывают множество, состоящее из всех элементов множества Уравнения по комбинаторике с решениямине принадлежащих множеству Уравнения по комбинаторике с решениямиЕго обозначают Уравнения по комбинаторике с решениямиНапример, если Уравнения по комбинаторике с решениямиУравнения по комбинаторике с решениями

Говоря «множество», «подмножество», порядок их элементов не учитывают. Говорят, что они не упорядочены. Рассматривают и упорядоченные множества. Так называют множества с фиксированным порядком элементов. Их обозначают не фигурными, а круглыми скобками. Например, из элементов множества Уравнения по комбинаторике с решениямиможно образовать 6 трёхэлементных упорядоченных множеств: Уравнения по комбинаторике с решениями

Как множества, все они равны, как упорядоченные множества — разные.

Существуют задачи, в которых надо определить, сколько различных подмножеств или упорядоченных подмножеств можно образовать из элементов данного множества. Их называют комбинаторными задачами, а раздел математики, в котором рассматривается решение комбинаторных задач, называют комбинаторикой.

Комбинаторика — раздел математики, посвящённый решению задач выбора и расположения элементов некоторого конечного множества в соответствии с заданными правилами.

Рассмотрим два основных правила, с помощью которых решается много комбинаторных задач.

Пример №17

В городе Уравнения по комбинаторике с решениямиесть два университета — политехнический и экономический. Абитуриенту нравятся три факультета в политехническом университете и два — в экономическом. Сколько возможностей имеет студент для поступления в университет?

Решение:

Обозначим буквой Уравнения по комбинаторике с решениямимножество факультетов, которые выбрал абитуриент в политехническом университете, а буквой Уравнения по комбинаторике с решениями— в экономическом: Уравнения по комбинаторике с решениямиПоскольку эти множества не имеют общих элементов, то в делом абитуриент имеет Уравнения по комбинаторике с решениямивозможностей для поступления в университет.

Описанную ситуацию можно обобщить в виде утверждения, которое называется правилом суммы.

Если элемент некоторого множества Уравнения по комбинаторике с решениямиможно выбрать Уравнения по комбинаторике с решениямиспособами, а элемент множества Уравнения по комбинаторике с решениямиспособами, то элемент из множества Уравнения по комбинаторике с решениямиили из множества Уравнения по комбинаторике с решениямиможно выбрать Уравнения по комбинаторике с решениямиспособами.

Правило суммы распространяется и на большее количество множеств.

Пример №18

Планируя летний отдых, семья определилась с местами его проведения: в Одессе — 1, в Евпатории — 3, в Ялте — 2, в Феодосии — 2. Сколько возможностей выбора летнего отдыха имеет семья?

Решение:

Поскольку все базы отдыха разные, то для решения задачи достаточно найти сумму элементов всех множеств, о которых говорится: Уравнения по комбинаторике с решениямиСледовательно, семья может выбирать отдых из 8 возможных.

Пример №19

От пункта Уравнения по комбинаторике с решениямидо пункта Уравнения по комбинаторике с решениямиведут три тропинки, а от Уравнения по комбинаторике с решениями— две. Сколько маршрутов можно проложить от пункта Уравнения по комбинаторике с решениямидо пункта Уравнения по комбинаторике с решениями

Решение:

Чтобы пройти от пункта Уравнения по комбинаторике с решениямидо пункта Уравнения по комбинаторике с решенияминадо выбрать одну из трёх тропинок: 1, 2 или 3 (рис. 136). После этого следует выбрать одну из двух других троп: 4 или 5. Всего от пункта Уравнения по комбинаторике с решениямидо пункта Уравнения по комбинаторике с решениямиведут 6 маршрутов, потому что Уравнения по комбинаторике с решениямиВсе эти маршруты можно обозначить с помощью пар:Уравнения по комбинаторике с решениями

Обобщим описанную ситуацию.

Если первый компонент пары можно выбрать Уравнения по комбинаторике с решениямиспособами, а . второй — Уравнения по комбинаторике с решениямиспособами, то такую пару можно выбрать Уравнения по комбинаторике с решениямиспособами.

Это — правило произведения, его часто называют основным правилом комбинаторики. Обратите внимание: речь идёт об упорядоченных парах, составленных из различных компонентов.

Правило произведения распространяется и на упорядоченные тройки, четвёрки и любые другие упорядоченные конечные множества. В частности, если первый компонент упорядоченной тройки можно выбрать Уравнения по комбинаторике с решениямиспособами, второй — Уравнения по комбинаторике с решениямиспособами, третий — Уравнения по комбинаторике с решениямиспособами, то такую упорядоченную тройку можно выбрать Уравнения по комбинаторике с решениямиспособами. Например, если столовая на обед приготовила 2 первых блюда — борщ (б) и суп (с ), 3 вторых — котлеты (к), вареники (в), голубцы (г) и 2 десертных — пирожные (п) и мороженое (м), то всего из трёх блюд столовая может предложить 12 различных наборов, поскольку Уравнения по комбинаторике с решениями

Уравнения по комбинаторике с решениями

Уравнения по комбинаторике с решениями

Описанной ситуации соответствует диаграмма, изображённая на рисунке 137. Такие диаграммы называют деревьями.

Пример №20

Сколько разных поездов можно составить из 6 вагонов, если каждый из вагонов можно поставить на любом месте?

Решение:

Первым можно поставить любой из б вагонов. Имеем 6 выборов. Второй вагон можно выбрать из оставшихся 5 вагонов. Поэтому, согласно правилу умножения, два первых вагона можно выбрать Уравнения по комбинаторике с решениямиспособами. Третий вагон можно выбрать из 4 вагонов, которые остались. Поэтому три первых вагона можно выбрать Уравнения по комбинаторике с решениямиспособами. Продолжая подобные рассуждения, приходим к ответу: всего можно составить Уравнения по комбинаторике с решениямиразличных поездов.

Обратите внимание на решение последней задачи. Оно свелось к вычислению произведения всех натуральных чисел от 1 до 6. В комбинаторике подобные произведения вычисляют часто.

Произведение всех натуральных чисел от 1 до Уравнения по комбинаторике с решенияминазывают Уравнения по комбинаторике с решениямифакториалом и обозначают Уравнения по комбинаторике с решениями

Уравнения по комбинаторике с решениями

Условились считать, что Уравнения по комбинаторике с решениями

Языком теории множеств правила суммы и произведения можно сформулировать следующим образом.

Если пересечение множеств Уравнения по комбинаторике с решениямипустое, то количество элементов в их объединении Уравнения по комбинаторике с решениямиравно сумме количества элементов множеств Уравнения по комбинаторике с решениями

Уравнения по комбинаторике с решениями

Если множества Уравнения по комбинаторике с решениямиимеют общие элементы, то

Уравнения по комбинаторике с решениями

Если множества Уравнения по комбинаторике с решениямиконечны, то количество возможных пар Уравнения по комбинаторике с решениямиравно произведению количества элементов множеств Уравнения по комбинаторике с решениями

Уравнения по комбинаторике с решениями

Пример №21

В розыгрыше на первенство города по баскетболу принимают участие команды из 12 школ. Сколькими способами могут быть распределены первое и второе места?

Решение:

Первое место может получить одна из 12 команд. После того, как определён обладатель первого места, второе место может получить одна из 11 команд. Следовательно, общее количество способов, которыми можно распределить первое и второе места, равно Уравнения по комбинаторике с решениями

Пример №22

Сколько четырёхзначных чисел можно составить из цифр 0,1, 2, 3, 4, 5, если ни одна цифра не повторяется?

Решение:

Первой цифрой числа может быть одна из 5 цифр 1, 2, 3, 4, 5. Если первая цифра выбрана, то вторая может быть выбрана 5-ю способами, третья — 4-мя, четвёртая — 3-мя. Согласно правилу умножения общее число способов равно:

Уравнения по комбинаторике с решениями

Пример №23

Упростите выражение Уравнения по комбинаторике с решениями

Решение:

Уравнения по комбинаторике с решениямиУравнения по комбинаторике с решениями

Размещения и перестановки

Задача:

Сколькими способами собрание из 20 человек может избрать председателя и секретаря?

Решение:

Председателя можно выбрать 20-ю способами, секретаря — из остальных 19 человек — 19-ю способами. По правилу произведения председателя и секретаря собрания могут выбрать Уравнения по комбинаторике с решениямиспособами.

Обобщим задачу. Сколько упорядоченных Уравнения по комбинаторике с решениямиэлементных подмножеств можно составить из Уравнения по комбинаторике с решениямиразличных элементов? На первое место можно поставить любой из данных Уравнения по комбинаторике с решениямиэлементов. На второе место — любой из остальных Уравнения по комбинаторике с решениямиэлементов и т. д. На последнее Уравнения по комбинаторике с решениямиместо можно поставить любой из остальных Уравнения по комбинаторике с решениямиэлементов. Из правила произведения следует, что из данных Уравнения по комбинаторике с решениямиэлементов можно получить Уравнения по комбинаторике с решениямиУравнения по комбинаторике с решениями-элементных упорядоченных подмножеств.

Например, из 4 элементов Уравнения по комбинаторике с решениямиупорядоченных двухэлементных подмножеств можно образовать всего Уравнения по комбинаторике с решениямиУравнения по комбинаторике с решениями

Упорядоченое Уравнения по комбинаторике с решениями-элементное подмножество Уравнения по комбинаторике с решениямиэлементного множества называют размещением из Уравнения по комбинаторике с решениямиэлементов Уравнения по комбинаторике с решениями Их число обозначают Уравнения по комбинаторике с решениями

Из предыдущих рассуждений следует, что Уравнения по комбинаторике с решениямии что для любых натуральных Уравнения по комбинаторике с решениями

Уравнения по комбинаторике с решениями

В правой части этого равенства Уравнения по комбинаторике с решениямимножителей. Поэтому результат можно сформулировать в виде такого утверждения.

Число размещений из Уравнения по комбинаторике с решениямиэлементов по Уравнения по комбинаторике с решениямиравно произведению Уравнения по комбинаторике с решениямипоследовательных натуральных чисел, наибольшее из которых Уравнения по комбинаторике с решениями

Примеры:

Уравнения по комбинаторике с решениями

Пример №24

Сколькими способами можно составить дневное расписание из пяти разных уроков, если класс изучает 10 различных предметов?

Решение:

Речь идёт об упорядоченных 5-элементных подмножествах некоторого множества, состоящего из 10 элементов.

Это размещения. Уравнения по комбинаторике с решениями

Ответ. 30 240 способами.

Число размещений из Уравнения по комбинаторике с решениямиэлементов по Уравнения по комбинаторике с решениямиможно вычислять и по другой формуле: Уравнения по комбинаторике с решениями(проверьте самостоятельно).

Размещение Уравнения по комбинаторике с решениямиэлементов по Уравнения по комбинаторике с решенияминазывают перестановками из Уравнения по комбинаторике с решениямиэлементов. Их число обозначают Уравнения по комбинаторике с решениями

Например, из трёх элементов Уравнения по комбинаторике с решениямиможно образовать 6 различных перестановок: Уравнения по комбинаторике с решениямиСледовательно, Уравнения по комбинаторике с решениями

Подставив в формулу числа размещений Уравнения по комбинаторике с решениямиполучим, что Уравнения по комбинаторике с решениями

Число перестановок из Уравнения по комбинаторике с решениямиэлементов равно Уравнения по комбинаторике с решениями!

Примеры:

Уравнения по комбинаторике с решениями

Пример №25

Сколькими способами можно составить список из 10 фамилий?

Решение:

Уравнения по комбинаторике с решениями

Ответ. 3 628 800 способами.

Некоторые комбинаторные задачи сводятся к решению уравнений, в которых переменная указывает на количество элементов в некотором множестве или подмножестве. Рассмотрим несколько таких уравнений.

Пример №26

Решите уравнение Уравнения по комбинаторике с решениями

Решение:

Пользуясь формулой размещений, данное уравнение можно заменить таким:

Уравнения по комбинаторике с решениями

По условию задачи Уравнения по комбинаторике с решениями— натуральное число, поэтому Уравнения по комбинаторике с решениями— посторонний корень. Следовательно, Уравнения по комбинаторике с решениями

Пример №27

Решите уравнение Уравнения по комбинаторике с решениями

Решение:

Запишем выражения Уравнения по комбинаторике с решениямичерез произведения.

Имеем: Уравнения по комбинаторике с решениями

Поскольку по смыслу задачи Уравнения по комбинаторике с решениямиПоэтому последнее уравнение можно сократить на произведение Уравнения по комбинаторике с решениямиТогда Уравнения по комбинаторике с решениями Уравнения по комбинаторике с решениямиНо уравнение Уравнения по комбинаторике с решениямиудовлетворяет только одно значение: Уравнения по комбинаторике с решениями

Пример №28

Команда из трёх человек выступает в соревнованиях по художественной гимнастике, в которых принимают участие ещё 27 спортсменок. Сколькими способами могут распределиться места между членами команды, при условии, что на этих соревнованиях ни одно место не делится?

Решение:

Речь идёт об упорядоченных 3-элементных подмножествах множества, состоящего из 30 элементов. Это — размещения. Уравнения по комбинаторике с решениями

Пример №29

Сколькими способами можно разместить на полке 5 дисков?

Решение:

Речь идёт об упорядоченных 5-элементных множествах. Искомое количество способов равно Уравнения по комбинаторике с решениями

Ответ. 120 способами.

Пример №30

Изображённое на рисунке 140 кольцо раскрашено в 7 цветов. Сколько существует таких колец, раскрашенных теми же цветами только в других последовательностях?

Решение:

Зафиксируем одну какую-нибудь часть кольца, окрашенную одним цветом, б других частей можно раскрасить Уравнения по комбинаторике с решениямиспособами.

Уравнения по комбинаторике с решениями

Ответ. 720 колец.

Пример №31

Сколько можно составить различных неправильных дробей, числителями и знаменателями которых есть числа 3,5, 7,9,11,13?

Решение:

Способ 1. Дробей, у которых числитель не равен знаменателю, можно составить Уравнения по комбинаторике с решениямито есть Уравнения по комбинаторике с решениямиИз этих дробей только половина — неправильных, то есть — 15.

Неправильными являются также дроби, у которых числитель равен знаменателю. Таких дробей в нашем случае 6. Итак, всего можно составить Уравнения по комбинаторике с решениями(дробь).

Способ 2. Если знаменатель неправильной дроби 3, то его числителями могут быть все 6 данных чисел. Если знаменатель 5, то числителями неправильной дроби могут быть 5 чисел (5, 7, 9, 11, 13) и т.д. Наконец, если знаменатель — число 13, то существует только 1 неправильная дробь, со знаменателем 13. Всего таких неправильных дробей существует Уравнения по комбинаторике с решениями

Уравнения по комбинаторике с решениями

Комбинации и бином ньютона

Пусть дано множество из трёх элементов: Уравнения по комбинаторике с решениямиЕго двухэлементных подмножеств (не упорядоченных) существует всего три: Уравнения по комбинаторике с решениямиГоворят, что существует 3 комбинации из трёх элементов по два. Пишут: Уравнения по комбинаторике с решениями

Комбинацией из Уравнения по комбинаторике с решениями элементов по Уравнения по комбинаторике с решениями называют любое Уравнения по комбинаторике с решениямиэлементное подмножество Уравнения по комбинаторике с решениямиэлементного множества.

Число комбинаций из Уравнения по комбинаторике с решениямиэлементов по Уравнения по комбинаторике с решениямиобозначают Уравнения по комбинаторике с решениямиВ отличие от размещений, комбинации — подмножества неупорядоченные.

Сравните: Уравнения по комбинаторике с решениямиПри тех же значениях Уравнения по комбинаторике с решениямизначение Уравнения по комбинаторике с решениямименьше Уравнения по комбинаторике с решениямиМожно также указать, во сколько раз меньше. Каждую Уравнения по комбинаторике с решениямиэлементную комбинацию можно упорядочить Уравнения по комбинаторике с решениямиспособами. В результате из одной комбинации получают Уравнения по комбинаторике с решениямиразмещений (упорядоченных подмножеств) из тех же элементов. Итак,

число Уравнения по комбинаторике с решениямиэлементных комбинаций в Уравнения по комбинаторике с решениямираз меньше числа размещений из тех же Уравнения по комбинаторике с решениямиэлементов.

То есть, Уравнения по комбинаторике с решениямиотсюда

Уравнения по комбинаторике с решениями

Пример №32

Вычислите: Уравнения по комбинаторике с решениями

Решение:

Уравнения по комбинаторике с решениями

Обратите внимание! Уравнения по комбинаторике с решениямиПолагают также, что Уравнения по комбинаторике с решениямидля любого Уравнения по комбинаторике с решениями

Пример №33

Сколькими способами из 25 учеников можно выбрать на конференцию двух делегатов?

Решение:

Здесь Уравнения по комбинаторике с решениямипорядок учеников не имеет значения.

Уравнения по комбинаторике с решениями

Ответ. 300-ми способами.

Докажем, что для натуральных значений Уравнения по комбинаторике с решениямиправильно тождество Уравнения по комбинаторике с решениями

Доказательство. Пусть дано Уравнения по комбинаторике с решениямиразличных элементов: Уравнения по комбинаторике с решениямиВсего из них можно образовать Уравнения по комбинаторике с решениямиразличных Уравнения по комбинаторике с решениямиэлементных комбинаций. Это количество комбинаций вычислим другим способом. Из данных Уравнения по комбинаторике с решениямиэлементов, кроме последнего Уравнения по комбинаторике с решениямиможно образовать Уравнения по комбинаторике с решениямикомбинаций. Остальные Уравнения по комбинаторике с решениямиэлементные комбинации из всех данных элементов можно образовать, если к каждой комбинации из первых Уравнения по комбинаторике с решениямиэлементов по Уравнения по комбинаторике с решениямидописать элемент Уравнения по комбинаторике с решениямиТаких комбинаций Уравнения по комбинаторике с решениями

Следовательно, Уравнения по комбинаторике с решениямиА это и требовалось доказать.

Такое комбинаторное тождество можно доказать также, воспользовавшись формулой числа комбинаций.

С комбинациями тесно связана формула бинома Ньютона. Вспомните формулу квадрата двучлена: Уравнения по комбинаторике с решениями

Умножив Уравнения по комбинаторике с решениямиполучим формулы:

Уравнения по комбинаторике с решениями

Эти три формулы можно записать и так:

Уравнения по комбинаторике с решениями

Оказывается, для каждого натурального значения Уравнения по комбинаторике с решениямиправильна и общая формула:

Уравнения по комбинаторике с решениями

Это тождество называют формулой бинома Ньютона. а её правую часть разложением бинома Ньютона. Бином — латинское название двучлена. Пользуясь этой формулой, возведём, например, двучлен Уравнения по комбинаторике с решениямив пятую степень. Поскольку Уравнения по комбинаторике с решениями

Уравнения по комбинаторике с решениями

Доказать формулу бинома Ньютона можно методом математической индукции.

Доказательство. Предположим, что формула Уравнения по комбинаторике с решениямиверна для некоторого натурального показателя степени Уравнения по комбинаторике с решениямиПокажем, что тогда она верна и для следующего за ним значения Уравнения по комбинаторике с решениями

Уравнения по комбинаторике с решениями

Выражения в скобках преобразованы согласно формулы

Уравнения по комбинаторике с решениями

Следовательно, если формула бинома Ньютона верна для Уравнения по комбинаторике с решениямито она правильна и для Уравнения по комбинаторике с решениямиДля Уравнения по комбинаторике с решениямиона правильна, так как Уравнения по комбинаторике с решениямиПоэтому на основе аксиомы математической индукции можно утверждать, что формула верна для любого натурального показателя Уравнения по комбинаторике с решениями

Вычислять коэффициенты разложения бинома Ньютона можно не по формуле числа комбинаций, а пользуясь числовым треугольником Паскаля — своеобразным способом вычисления коэффициентов разложения бинома Ньютона Уравнения по комбинаторике с решениями

Уравнения по комбинаторике с решениями

Треугольник Паскаля можно продолжать как угодно далеко. Это следует из тождества Уравнения по комбинаторике с решениямиЕго крайние числа — единицы, а каждое другое равно сумме двух ближайших к нему чисел сверху.

Например, прибавляя числа шестой строки (для Уравнения по комбинаторике с решениямиполучим числа следующей строки (для Уравнения по комбинаторике с решениямиСледовательно, Уравнения по комбинаторике с решениямиОбщий член разложения бинома Уравнения по комбинаторике с решениямиможно определить по формуле Уравнения по комбинаторике с решениями

  • первый член — Уравнения по комбинаторике с решениями
  • второй член — Уравнения по комбинаторике с решениями
  • третий член — Уравнения по комбинаторике с решениями

Пример №34

В турнире по шашкам приняли участие 5 девушек и 7 юношей. Каждый участник сыграл один раз с каждым другим. Сколько партий было: а) между девушками; б) между юношами; в) между юношами и девушками?

Решение:

а) Речь идёт о 2-элементных подмножествах (неупорядоченных) множества, состоящего из 5 элементов. Это — комбинации. Уравнения по комбинаторике с решениями

б) Аналогично Уравнения по комбинаторике с решениями

в) Воспользуемся правилом умножения. Поскольку каждой из 5 девушек предстоит сыграть с каждым из 7 юношей, возможных случаев Уравнения по комбинаторике с решениями

Пример №35

Для дежурства в столовой приглашают 3-х учеников из 7 класса и 2-х учеников из 10 класса. Сколькими способами это можно сделать, если в 7 классе учится 24 ученика, а в 10 классе — 18.

Решение:

Речь идёт о неупорядоченных подмножествах двух разных множеств. Это — комбинации.
Уравнения по комбинаторике с решениями
По правилу произведения имеем Уравнения по комбинаторике с решениямиспособов выбрать учащихся для дежурства.

Пример №36

Сколько разных делителей имеет число 1001?

Решение:

Разложим заданное число на простые множители: Уравнения по комбинаторике с решениямиЕсли число Уравнения по комбинаторике с решениями— делитель числа 1001, то оно должно быть одним из чисел 7, 11,13 (три случая) или любым их произведением. Различных произведений может быть Уравнения по комбинаторике с решениямиДелителем данного числа есть ещё единица. Следовательно, число 1001 имеет Уравнения по комбинаторике с решениямиделителей.

Пример №37

Докажите, что выпуклый Уравнения по комбинаторике с решениямиугольник имеет Уравнения по комбинаторике с решениямидиагоналей.

Решение:

Отрезков, концами которых являются Уравнения по комбинаторике с решениямивершин данного Уравнения по комбинаторике с решениями-угольника, существует Уравнения по комбинаторике с решениямиСреди них есть и Уравнения по комбинаторике с решениямисторон данного Уравнения по комбинаторике с решениями-угольника. Поэтому диагоналей он имеет Уравнения по комбинаторике с решениямиУравнения по комбинаторике с решениями

Пример №38

Уравнения по комбинаторике с решениями

Решение:

Уравнения по комбинаторике с решениями

Уравнения по комбинаторике с решениями

Все члены разложения бинома Ньютона Уравнения по комбинаторике с решениямитакие же, как и члены разложения бинома Уравнения по комбинаторике с решениямитолько их члены с чётными номерами отрицательные.

Пример №39

Найдите номер члена разложения Уравнения по комбинаторике с решениямикоторый не содержит Уравнения по комбинаторике с решениями

Решение:

Воспользуемся формулой общего члена разложения бинома. Имеем:

Уравнения по комбинаторике с решениями

По условию задачи Уравнения по комбинаторике с решениямито есть Уравнения по комбинаторике с решениямиОтсюда Уравнения по комбинаторике с решениямиСледовательно, не содержит Уравнения по комбинаторике с решениямишестой член разложения бинома.

Видео:Математика без Ху!ни. Теория вероятностей, комбинаторная вероятность.Скачать

Математика без Ху!ни. Теория вероятностей, комбинаторная вероятность.

Элементы комбинаторики

Решение многих задач теории вероятностей требует знания элементов комбинаторики, основными понятиями которой являются перестановки, размещения и сочетания.

Определение: Перестановки — это комбинации из одних и тех же элементов, отличающиеся только порядком элементов.

Пример:

Даны три числа 1, 2, 3. Определить количество комбинаций из этих элементов, отличающиеся только порядком элементов.

Решение:

Комбинации из данных элементов, отличающиеся только порядком элементов: 123; 132; 213; 231; 321; 312. Всего таких комбинаций Уравнения по комбинаторике с решениямиЕсли дано n элементов, то число перестановок Уравнения по комбинаторике с решениямиO2. Размещения — это комбинации, составленные из n различных элементов по m элементов, которые отличаются либо составом элементов, либо их расположением.

Пример:

Даны три числа 1, 2, 3. Определить количество размещений из этих элементов по два, отличающиеся составом или порядком элементов.

Решение:

Комбинации из данных элементов по два, отличающиеся составом или порядком элементов: 12; 21; 23; 32; 13; 31. Всего таких комбинаций 6. Если дано n элементов, то число размещений по m элементов, которые отличаются либо составом элементов, либо их расположением: Уравнения по комбинаторике с решениями

Определение: Сочетания — это комбинации, составленные из n различных элементов по m элементов, которые отличаются друг от друга хотя бы одним элементом.

Пример:

Даны три числа 1, 2, 3. Определить количество размещений из этих элементов по два, отличающиеся хотя бы одним элементом.

Решение:

Комбинации из данных элементов по два, отличающиеся хотя бы одним элементом: 12; 23; 13. Всего таких комбинаций 3. Если дано n элементов, то число сочетаний по m элементов, которые отличаются хотя бы одним элементом:Уравнения по комбинаторике с решениями

Пример:

Пусть в урне находится n прономерованных шаров. Определить количество способов, которыми можно извлечь из урны эти шары один за другим.

Решение:

Число способов равно числу различных комбинаций из п элементов, отличающихся только порядком элементов, т.е. числу перестановок: Уравнения по комбинаторике с решениями

Пример:

Из колоды, содержащей 36 карт, наугад вынимают 3 карты. Найти вероятность того, что среди выбранных карт окажется один туз.

Решение:

Событие А состоит в том, что среди выбранных карт окажется один туз. Это сложное событие состоит из двух событий: выбирается один туз из четырех, а две другие карты выбираются из оставшихся 32 карт. Следовательно, число случаев, благоприятствующих появлению события A, равно Уравнения по комбинаторике с решениямиВсего возможных равновероятных исходов, образующих полную группу определяется числом сочетаний из 36 карт по 3 карты, т.е. Уравнения по комбинаторике с решениямиТаким образом, вероятность события А равна Уравнения по комбинаторике с решениями

Арифметика случайных событий

Будем считать, что все события, которые могут произойти в рамках данного эксперимента, располагаются внутри квадрата G, тогда невозможные события располагаются вне квадрата G (Рис. 2): Уравнения по комбинаторике с решениями

Рис. 2. Квадрат возможных событий.

Таким образом, достоверное событие определяется внутренней частью квадрата, а невозможное — областью вне квадрата.

Определение: Суммой двух случайных событий А и В называется третье случайное событие С, которое состоит в том, что произойдет (или не произойдет) или событие А, или событие В : С = А + В (Рис. 3).

Определение: Суммой n случайных событий Уравнения по комбинаторике с решенияминазывается случайное событие С, которое реализуется в данном опыте, если произойдет (или не произойдет) или одно событий Уравнения по комбинаторике с решениями, или любая их совокупность: Уравнения по комбинаторике с решениями

Уравнения по комбинаторике с решениями

Рис. 3. Сумма случайных событий

Замечание: Если в словесном описании сложного события присутствует разделительный союз “или” между элементарными событиями, то речь идет о сумме этих элементарных событий.

Замечание: Суммой события А и ему противоположного события Уравнения по комбинаторике с решениямиявляется достоверное событие Уравнения по комбинаторике с решениямит.е. Уравнения по комбинаторике с решениямиСледовательно, противоположное событие можно записать в виде Уравнения по комбинаторике с решениями

Определение: Произведением двух случайных событий А и В называется третье случайное событие С, которое состоит в том, что произойдет (или не произойдет) и событие А, и событие В : Уравнения по комбинаторике с решениями(Рис. 4). Уравнения по комбинаторике с решениями

Рис. 4. Произведение случайных событий.

Определение: Произведением n случайных событий Уравнения по комбинаторике с решенияминазывается случайное событие С, которое реализуется в данном опыте, если произойдет (или не произойдет) совместная реализация событий Уравнения по комбинаторике с решениями

Замечание: Если в словесном описании сложного события присутствует соединительный союз “и” между элементарными событиями, то речь идет о произведении этих элементарных событий.

Пример №40

Пусть имеются передатчик и приемник. Приемник удален от передатчика недостаточно большое расстояние, при котором он может при определенных условиях не принять один из сигналов, переданных передатчиком. Пусть передатчик послал три сигнала. Определить следующие сложные события:

  • а) приемник принят только второй сигнал (событие А );
  • б) приемник принял только один сигнал (событие В);
  • в) приемник принял не менее двух сигналов (2 или 3 сигнала — событие С);
  • г) приемник не принял ни одного сигнала (событие D);
  • д) приемник принял хотя бы один сигнал (событие E).

Решение:

Обозначим через Уравнения по комбинаторике с решениямиэлементарное событие, состоящее в том, что приемник принял сигнал i.

Сложное событие А состоит в том, что приемник не принял первый сигнал и принял второй сигнал, и не принял третий сигнал. Так как между элементарными событиями стоит соединительный союз “и”, то речь идет о их произведении, т.е. Уравнения по комбинаторике с решениями

Сложное событие В состоит в том, что приемник принял или первый сигнал, или принял второй сигнал, или принял третий сигнал. Так как между элементарными событиями стоит разделительный союз “или”, то речь идет о сумме сложных событии, т.е. Уравнения по комбинаторике с решениями

Рассуждая аналогично, получим выражения для остальных событий: Уравнения по комбинаторике с решениямиСложное событие Е содержит в своем словесном описании слова “хотя бы один”, следовательно, оно противоположно событию, содержащему в своем словесном описании слова “ни один”, т.е. событию D: Уравнения по комбинаторике с решениями

Теорема сложения вероятностей несовместных событий

Теорема: Если случайные события А и В несовместны, то вероятность их суммы равна сумме вероятностей этих событий, т.е. Р(А + В) = Р(А) + Р(В)

Доказательство: Пусть в данном опыте имеется n равновозможных, элементарных, несовместных событий и пусть в m случаях наступает событие А, а в l случаях-событие В. Тогда появлению события А + В благоприятствует m+l исходов. Поэтому Уравнения по комбинаторике с решениями

Следствие: Если имеется N событий, то Уравнения по комбинаторике с решениями

Следствие: Если события Уравнения по комбинаторике с решениями(Уравнения по комбинаторике с решениями) образуют полную группу, то Уравнения по комбинаторике с решениями

Доказательство: Так как события Уравнения по комбинаторике с решениямиобразуют полную группу равно возможных, элементарных, несовместных событий, то их сумма есть достоверное событие Уравнения по комбинаторике с решениямиа вероятность достоверного события равна 1.

Следствие: Вероятность суммы противоположных событий равна 1.

Доказательство: В силу того, что события А и ему противоположное событие Уравнения по комбинаторике с решениямиобразуют полную группу несовместных событий, то по следствию вероятность их суммы равна 1.

Замечание: Если сложное событие состоит из суммы элементарных событий, то перед применением теоремы надо определить совместны или несовместны элементарные события.

Пример:

Пусть в урне находится 5 белых шаров, 3 — красных и 4 — зеленых. Из урны наудачу вынули шар. Какова вероятность того, что данный шар цветной?

Решение:

Событие, состоящее в том, что из урны извлечен красный шар, обозначим через А. Событие, состоящее в том, что из урны извлечен зеленый шар, обозначим через В. Тогда извлечение цветного шара есть событие С. Так как события А и В несовместны, т.е. событие С состоит в том, что из урны извлечен или событие А , или событие В, то С = А + В. Используя теорему о сложении вероятностей несовместных событий, получим:

Уравнения по комбинаторике с решениями

Зависимые и независимые события. Условная и безусловная вероятности

Определение: Случайные события А и В называются независимыми, если появление одного из них не влияет на вероятность появления другого события, в противном случае события называются зависимыми.

Замечание: В этом определении речь идет не о причинно-следственной связи между событиями, а о вероятностной (появление одного из них не влияет на вероятность появления другого события), которая является более общей зависимостью между событиями.

Пример №41

В хранилище находится 10 исправных и 5 неисправных приборов, причем неизвестно, какие из них исправные, а какие — нет. Обозначим событием А — из хранилища взят исправный прибор, а В — взят неисправный прибор. Пусть вначале взят неисправный прибор. Определить вероятности указанных событий с возвращением неисправного прибора на склад и без возвращения неисправного прибора в хранилище.

Решение:

Если неисправный прибор возвращается в хранилище, то события А и В независимы и их вероятности равны Уравнения по комбинаторике с решениямиВо втором случае, когда неисправный прибор не возвращается на склад, общее количество приборов в хранилище изменилось и стало равным 14, причем неисправных приборов будет храниться 4. Следовательно, произошедшее событие В изменило вероятности события А и В: Уравнения по комбинаторике с решениямит.е. при такой организации эксперимента события А и В являются зависимыми.

Определение: Вероятность случайного события называется безусловной, если при ее вычислении на комплекс условий, в которых рассматривается это случайное событие, не накладывается никаких дополнительных ограничений. Безусловная вероятность обозначается Уравнения по комбинаторике с решениями

Определение: Вероятность случайного события называется условной, если она вычисляется при условии, что произошло другое случайное событие. Условная вероятность обозначается Уравнения по комбинаторике с решениями

Теорема умножения вероятностей

Т.2. Вероятность совместного появления двух случайных событий А и В равна произведению вероятности одного из них на условную вероятность другого события, вычисленную при условии, что первое событие имело место: Уравнения по комбинаторике с решениями

Доказательство: Пусть событие А состоит в том, что брошенная точка наугад в квадрат G попадает в область А, которая имеет площадь Уравнения по комбинаторике с решениямиСобытие В состоит в том, что брошенная наугад в квадрат G точка попадает в область В с площадью Уравнения по комбинаторике с решениямиПусть весь квадрат имеет площадь S, а область совместного наступления событий Уравнения по комбинаторике с решениямиимеет площадь Уравнения по комбинаторике с решениями(Рис. 5). Тогда вероятность события А равна Уравнения по комбинаторике с решениямиа события В — Уравнения по комбинаторике с решениями

Уравнения по комбинаторике с решениями

Рис. 5. Совместное наступление зависимых и независимых случайных событий.

Вероятность совместного наступления событий Уравнения по комбинаторике с решениями.Условные вероятности того, что произойдут указанные события, определяются по формулам: Уравнения по комбинаторике с решениямиТаким образом, можно записать, что вероятность совместного наступления событий Уравнения по комбинаторике с решениямиравна:

Уравнения по комбинаторике с решениями

Замечание: Если события А и В независимы, то Уравнения по комбинаторике с решениямит.е. безусловная и условная вероятности равны между собой.

В связи с вышеприведенным замечанием теорема об умножении вероятностей независимых случайных событий имеет вид:

ТЗ. Вероятность совместного наступления независимых событий равна произведению вероятностей этих событий: Уравнения по комбинаторике с решениями

Замечание: Независимость случайных событий всегда взаимная. Если Уравнения по комбинаторике с решениямито по теореме Уравнения по комбинаторике с решениямиоткуда следует, чтоУравнения по комбинаторике с решениями

Следствие: Методом математической индукции теоремы легко обобщается на произведение N зависимых событий:

Уравнения по комбинаторике с решениямиа теорема — для независимых событий: Уравнения по комбинаторике с решениями

Замечание: Если сложное событие представляется в виде произведения элементарных событий, то при вычислении вероятности такого события надо определить, зависимы или независимы эти элементарные события.

Видео:02 Комбинаторика ЗадачиСкачать

02  Комбинаторика  Задачи

Что такое комбинаторика

Понятие множества и его элементов:

  • Элемент а принадлежит множеству АУравнения по комбинаторике с решениямиУравнения по комбинаторике с решениями
  • Элемент Уравнения по комбинаторике с решениямипринадлежит множеству Уравнения по комбинаторике с решениямиУравнения по комбинаторике с решениями
  • В множестве нет элементовУравнения по комбинаторике с решениямиУравнения по комбинаторике с решениями

Множество можно представить как совокупность некоторых объектов, объединенных по определенному признаку. В математике множество — одно из основных неопределяемых понятий. Каждый объект, принадлежащий множеству А, называется элементом этого множества. Множество, не содержащее ни одного элемента, называется пустым множеством и обозначается Уравнения по комбинаторике с решениями.

ПодмножествоУравнения по комбинаторике с решениями

Уравнения по комбинаторике с решениями

Если каждый элемент множества А является элементом множества В, то говорят, что множество А является подмножеством множества В,

и записывают так: Уравнения по комбинаторике с решениямиИспользуется также запись Уравнения по комбинаторике с решениямиесли множество А или является подмножеством множества В, или равно множеству В.

Уравнения по комбинаторике с решениями

Два множества называются равными, если каждый элемент первого множества является элементом второго множества и, наоборот, каждый элемент второго множества является элементом первого множества.

Пересечение множествУравнения по комбинаторике с решениями

Уравнения по комбинаторике с решениями

Пересечением множеств A и В называют их общую часть, то есть множество С всех элементов, принадлежащих как множеству А, так и множеству В

Объединение множеств Уравнения по комбинаторике с решениями

Уравнения по комбинаторике с решениями

Объединением множеств А и В называют множество С, состоящее из всех элементов, принадлежащих хотя бы одному из этих множеств (А или В)

Разность множеств Уравнения по комбинаторике с решениями

Уравнения по комбинаторике с решениями

Разностью множеств А и В называется множество С, которое состоит из всех элементов, принадлежащих множеству А и не принадлежащих множеству В

Уравнения по комбинаторике с решениями

Если все рассматриваемые множества являются подмножествами некоторого универсального множества U, то разность U А называется дополнением множества А. Другими словами, дополнением множества А называется множество, состоящее из всех элементов, не принадлежащих множеству А (но принадлежащих универсальному множеству).

Объяснение и обоснование:

Понятие множества

Одним из основных понятий, которые используются в математике, является понятие множества. Для него не дается определения. Можно пояснить, что множеством называют произвольную совокупность объектов, а сами объекты — элементами данного множества. Так, можно говорить о множестве учеников в классе (элементы — ученики), множестве дней недели (элементы — дни недели), множестве натуральных делителей числа 6 (элементы — числа 1, 2, 3, 6) и т. д.

В курсах алгебры и алгебры и начал анализа чаще всего рассматривают множества, элементами которых являются числа, и поэтому их называют числовыми множествами.

Как правило, множества обозначают прописными буквами латинского алфавита. Например, если множество М состоит из чисел 1; 2; 3, то его обозначают так: М = . Тот факт, что число 2 входит в это множество (является элементом данного множества М) записывается с помощью специального значка Уравнения по комбинаторике с решениямиследующим образом: Уравнения по комбинаторике с решениями; а то, что число 5 не входит в это множество (не является элементом данного множества), записывается так:Уравнения по комбинаторике с решениями

Можно рассматривать также множество, не содержащее ни одного элемента, — пустое множество.

Например: множество простых делителей числа 1 — пустое множество.

Для некоторых множеств существуют специальные обозначения. Так, пустое множество обозначается символомУравнения по комбинаторике с решениями, множество всех натуральных чисел — буквой N, множество всех целых чисел — буквой Z, множество всех рациональных чисел — буквой Q, а множество всех действительных чисел — буквой R.

Множества бывают конечными и бесконечными в зависимости от того, какое количество элементов они содержат. Так, множества А = и М = — конечные потому, что содержат конечное число элементов, а множества N, Z, Q, R — бесконечные.

Множества задают или с помощью перечисления их элементов (это можно сделать только для конечных множеств), или с помощью описания, когда задается правило (характеристическое свойство), которое позволяет определить, принадлежит или нет данный объект рассматриваемому множеству. Например, А = (множество задано перечислением элементов), В — множество четных целых чисел (множество задано характеристическим свойством элементов множества). Последнее множество иногда записывают так: Уравнения по комбинаторике с решениями— четное целое число> или так: Уравнения по комбинаторике с решениями— здесь после вертикальной черточки записано характеристическое свойство.

В общем виде запись множества с помощью характеристического свойства можно обозначить так: Уравнения по комбинаторике с решениями— характеристическое свойство. Например,Уравнения по комбинаторике с решениями

Равенство множеств

Пусть А — множество цифр трехзначного числа 312, то есть А = , а В — множество натуральных чисел, меньших четырех, то есть В = . Поскольку эти множества состоят из одних и тех же элементов, то они считаются равными. Это записывают так: А = В.

Для бесконечных множеств таким способом (сравнивая все элементы) установить их равенство невозможно. Поэтому в общем случае равенство множеств определяется следующим образом.

Два множества называются равными, если каждый элемент первого множества является элементом второго множества и, наоборот, каждый элемент второго множества является элементом первого множества.

Из приведенного определения равенства множеств следует, что в множестве одинаковые элементы не различаются. Действительно, например, = , поскольку каждый элемент первого множества (1 или 2) является элементом второго множества и, наоборот, каждый элемент второго множества (1 или 2) является элементом первого. Поэтому, записывая множество, чаще всего каждый его элемент записывают только один раз.

Подмножество

Если каждый элемент множества А является элементом множества В, то говорят, что множество А является подмножеством множества В.

Это записывают следующим образом: Уравнения по комбинаторике с решениями

Например, Уравнения по комбинаторике с решениями(поскольку любое натуральное число — целое), Уравнения по комбинаторике с решениями(поскольку любое целое число — рациональное), Уравнения по комбинаторике с решениями(поскольку любое рациональное число — действительное).

Полагают, что всегдаУравнения по комбинаторике с решениями, то есть пустое множество является подмножеством любого множества.

Иногда вместо записи Уравнения по комбинаторике с решениямииспользуется также запись Уравнения по комбинаторике с решениями, если множество А является подмножеством множества В или равно множеству В. Например, можно записать, что Уравнения по комбинаторике с решениями.

Сопоставим определение равенства множеств с определением подмножества. Если множества А и В равны, то: 1) каждый элемент множества А является элементом множества В, следовательно, А — подмножество ВУравнения по комбинаторике с решениями; 2) каждый элемент множества В является элементом множества А, следовательно, В — подмножество Уравнения по комбинаторике с решениямиТаким образом,

два множества равны, если каждое из них является подмножеством другого.

А = В означает то же, что Уравнения по комбинаторике с решениями

Иногда соотношения между множествами удобно иллюстрировать с помощью кругов (которые часто называют кругами Эйлера-Венна). Например, рисунок 118 иллюстрирует определение подмножества, а рисунок 119-отношения между множествами Уравнения по комбинаторике с решениями

Уравнения по комбинаторике с решениями

Операции над множествами

Над множествами можно выполнять определенные действия: находить их пересечение, объединение, разность. Дадим определение этих операций и проиллюстрируем их с помощью кругов.

Пересечением множеств А и В называют их общую часть, то есть множество С всех элементов, принадлежащих как множеству А, так и множеству В.

Пересечение множеств обозначают знаком Уравнения по комбинаторике с решениями(на рисунке 120 приведена иллюстрация и символическая запись определения пересечения множеств).

Например, если А = , В = , то Уравнения по комбинаторике с решениями

Объединением множеств А и В называют множество С, состоящее из всех элементов, принадлежащих хотя бы одному из этих множеств (А или В).

Объединение множеств обозначают знаком U (на рисунке 121 приведена иллюстрация и символическая запись определения объединения множеств).

Например, для множеств А и В из предыдущего примера Уравнения по комбинаторике с решениямиЕсли обозначить множество иррациональных чисел через М, то М U Q = R. Разностью множеств А и В называется множество С, состоящее из всех элементов, которые принадлежат множеству А и не принадлежат множеству В.

Разность множеств обозначают знаком . На рисунке 122 приведена иллюстрация и символическая запись определения разности множеств.

Например, если А = , В = , то АВ = , а В А = . Если В — подмножество А, то разность А В называют дополнением множества В до множества А (рис. 123).

Например, если обозначить множество иррациональных чисел через М, то R Q = М: множество М иррациональных чисел дополняет множество Q рациональных чисел до множества R всех действительных чисел.

Все множества, которые мы рассматриваем, являются подмножествами некоторого так называемого универсального множества U. Его обычно изображают в виде прямоугольника, а все остальные множества — в виде кругов внутри этого прямоугольника (рис. 124). Разность U А называется дополнением множества А. Уравнения по комбинаторике с решениями

Уравнения по комбинаторике с решениями

Дополнением множества А называется множество, состоящее из всехэлементов, не принадлежащих множеству А (но принадлежащих универсальному множеству U).

Дополнение множества А обозначается Уравнения по комбинаторике с решениями(можно читать: «А с чертой»). Например, если U = R и А = [0; 1], то Уравнения по комбинаторике с решениямиДля этого примера удобно использовать традиционную иллюстрацию множества действительных чисел на числовой прямой (рис. 125).

Видео:9 класс. Алгебра. Решение уравнений. Элементы комбинаторики.Скачать

9 класс. Алгебра. Решение уравнений. Элементы комбинаторики.

Комбинаторика и Бином Ньютона

Элементы комбинаторики:

Комбинаторика — раздел математики, в котором изучаются способы выбора и размещения элементов некоторого конечного множества на основании некоторых условий. Выбранные (или выбранные и размещенные) группы элементов называются Соединения с повторениямими.

Если все элементы полученного множества разные — получаем соединения без повторений, а если в полученном множестве элементы повторяются, то получаем соединения с повторениями*.

Перестановкой из п элементов называется любое упорядоченное множество из Уравнения по комбинаторике с решениямиэлементов.

Иными словами, это такое множество, для которого указано, какой элемент находится на первом месте, какой — на втором. какой — на п-м.

*Формулы для нахождения количества соединений с повторениями являются обязательными только для классов физико-математического профиля. Формула числа перестановок Уравнения по комбинаторике с решениями Уравнения по комбинаторике с решениями(читается: «Эн факториал»)

Количество различных шестизначных чисел, которые можно составить из цифр 1, 2, 3, 4, 5, 6, не повторяя эти цифры в одном числе, равно Уравнения по комбинаторике с решениями

Размещением из Уравнения по комбинаторике с решениямиэлементов по Уравнения по комбинаторике с решенияминазывается любое упорядоченное множество из Уравнения по комбинаторике с решениямиэлементов, состоящее из элементов Уравнения по комбинаторике с решениями-элементного множества Формула числа размещенийУравнения по комбинаторике с решениями

Уравнения по комбинаторике с решениями

Количество различных трехзначных чисел, которые можно составить из цифр 1,2,3, 4, 5, 6, если цифры не могут повторяться, равно Уравнения по комбинаторике с решениями

Сочетанием без повторений из Уравнения по комбинаторике с решениямиэлементов по Уравнения по комбинаторике с решенияминазывается любое Уравнения по комбинаторике с решениями-элементное подмножество Уравнения по комбинаторике с решениями-элементного множества Формула числа сочетанийУравнения по комбинаторике с решениями

Уравнения по комбинаторике с решениями(по определению считают, что Уравнения по комбинаторике с решениями)

Из класса, состоящего из 25 учащихся, можно выделить 5 учащихся для дежурства по школе Уравнения по комбинаторике с решениямиспособами, то есть Уравнения по комбинаторике с решениямиспособами. Некоторые свойства числа сочетаний без повторений Уравнения по комбинаторике с решениями

Схема решения комбинаторных задач

Если элемент А можно выбрать Уравнения по комбинаторике с решениямиспособами, а элемент В — Уравнения по комбинаторике с решениямиспособами, то А или В можно выбрать Уравнения по комбинаторике с решениямиспособами.

Если элемент А можно выбрать Уравнения по комбинаторике с решениямиспособами, а после этого элемент В — Уравнения по комбинаторике с решениямиспособами, то А и В можно выбрать Уравнения по комбинаторике с решениямиспособами. Выбор формулы

Учитывается ли порядок следования элементов в соединении?

Все ли элементы входят в соединение?

без повторений с повторениями без повторений с повторениями без повторений с повторениямиУравнения по комбинаторике с решениями

Объяснение и обоснование:

Понятие соединения

При решении многих практических задач приходится выбирать из определенной совокупности объектов элементы, имеющие те или иные свойства, размещать эти элементы в определенном порядке и т. д. Поскольку в этих задачах речь идет о тех или иных комбинациях объектов, то такие задачи называют комбинаторными. Раздел математики, в котором рассматриваются методы решения комбинаторных задач, называется комбинаторикой. В комбинаторике рассматривается выбор и размещение элементов некоторого конечного множества на основании определенных условий.

Выбранные (или выбранные и размещенные) группы элементов называют соединениями. Если все элементы полученного множества разные — получаем размещения без повторений, а если в полученном множестве элементы могут повторяться, то получаем размещения с повторениями. Рассматриваются соединения без повторений, а соединения с повторениями.

Решение многих комбинаторных задач базируется на двух основных правилах — правиле суммы и правиле произведения.

Правило суммы

Если на тарелке лежит 5 груш и 4 яблока, то выбрать один фрукт (то есть грушу или яблоко) можно 9 способами (5 + 4 = 9). В общем виде имеет место такое утверждение:

  • если элемент А можно выбрать Уравнения по комбинаторике с решениямиспособами, а элемент В — Уравнения по комбинаторике с решениямиспособами, то А или В можно выбрать Уравнения по комбинаторике с решениямиспособами.

Правило произведения

Если в киоске продают ручки 5 видов и тетради 4 видов, то выбрать набор из ручки и тетради (то есть пару — ручка и тетрадь) можно 5 • 4 = 20 способами (поскольку с каждой из 5 ручек можно взять любую из 4 тетрадей). В общем виде имеет место такое утверждение:

  • если элемент А можно выбрать m способами, а после этого элемент В — Уравнения по комбинаторике с решениямиспособами, то А и В можно выбрать m • п способами.

Это утверждение означает, что если для каждого из т элементов А можно взять в пару любой из Уравнения по комбинаторике с решениямиэлементов В, то количество пар равно произведению Уравнения по комбинаторике с решениями

Повторяя приведенные рассуждения несколько раз (или, иначе говоря, используя метод математической индукции), получаем, что правила суммы и произведения можно применять при выборе произвольного конечного количества элементов.

Следовательно, если приходится выбирать или первый элемент, или второй, или третий и т. д. элемент, количества способов выбора каждого еле-мента складывают, а когда приходится выбирать набор, в который входят и первый, и второй, и третий, и т. д. элементы, количества способов выбора каждого элемента перемножают.

Упорядоченные множества

При решении комбинаторных задач приходится рассматривать не только множества, в которых элементы можно записывать в любом порядке, но и так называемые упорядоченные множества. Для упорядоченных множеств существенным является порядок следования их элементов, то есть то, какой элемент записан на первом месте, какой на втором и т. д. В частности, если одни и те же элементы записать в разном порядке, то мы получим различные упорядоченные множества. Чтобы различить записи упорядоченного и неупорядоченного множеств, элементы упорядоченного множества часто записывают в круглых скобках, например Уравнения по комбинаторике с решениями

Рассматривая упорядоченные множества, следует учитывать, что упорядоченность не является свойством самого неупорядоченного множества (из которого мы получили упорядоченное), поскольку одно и то же множество можно по-разному упорядочить. Например, множество из трех чисел можно упорядочить по возрастанию: (-5; 1; 3), по убыванию: (3; 1; — 5), по возрастанию абсолютной величины числа: (1; 3; -5) и т. д.

Будем понимать, что для того чтобы задать конечное упорядоченное множество из п элементов, достаточно указать, какой элемент находится на первом месте, какой на втором, . какой на п-м.

Размещения

Размещением из Уравнения по комбинаторике с решениямиэлементов по Уравнения по комбинаторике с решенияминазывается любое упорядоченное множество из Уравнения по комбинаторике с решениямиэлементов, состоящее из элементов Уравнения по комбинаторике с решениями-элементного множества.

Например, из множества, содержащего три цифры , можно составить следующие размещения из двух элементов без повторений: (1;5),(1;7),(5; 7), (5; 1), (7; 1), (7; 5).

Количество размещений из Уравнения по комбинаторике с решениямиэлементов по Уравнения по комбинаторике с решениямиобозначается Уравнения по комбинаторике с решениями(читается: «А из Уравнения по комбинаторике с решениямипо Уравнения по комбинаторике с решениями», А — первая буква французского слова arrangement, что означает «размещение, приведение в порядок»). Как видим,Уравнения по комбинаторике с решениями

Выясним, сколько всего можно составить размещений из Уравнения по комбинаторике с решениямиэлементов по Уравнения по комбинаторике с решениямибез повторений. Составление размещения представим себе как последовательное заполнение Уравнения по комбинаторике с решениямимест, которые мы будем изображать в виде клеточек (рис. 126). На первое место мы можем выбрать один из п элементов заданного множества (то есть элемент для первой клеточки можно выбрать Уравнения по комбинаторике с решениямиспособами).

Если элементы нельзя повторять, то на второе место можно выбрать только один элемент из оставшихся, то есть из Уравнения по комбинаторике с решениями— 1 элементов. Теперь уже два элемента использованы и на третье место можно выбрать только один из Уравнения по комбинаторике с решениями— 2 элементов и т. д. На Уравнения по комбинаторике с решениями-e место можно выбрать только один из Уравнения по комбинаторике с решениямиэлементов.

Поскольку требуется выбрать элементы и на первое место, и на второе, . и наУравнения по комбинаторике с решениями-e, то используем правило произведения, получим следующую формулу числа размещений из Уравнения по комбинаторике с решениямиэлементов по Уравнения по комбинаторике с решениямиУравнения по комбинаторике с решениями

Например, Уравнения по комбинаторике с решениями(что совпадает с соответствующим значением, полученным выше). Аналогично можно обосновать формулу для нахождения числа размещений с повторениями.

При решении простейших комбинаторных задач важно правильно выбрать формулу, по которой будут проводиться вычисления. Для этого достаточно выяснить следующее: Уравнения по комбинаторике с решениями

  • — Учитывается ли порядок следования элементов в соединении?
  • — Все ли заданные элементы входят в полученное соединение?

Если, например, порядок следования элементов учитывается и из Уравнения по комбинаторике с решениямизаданных элементов в соединении используется только Уравнения по комбинаторике с решениямиэлементов, то по определению — это размещение из Уравнения по комбинаторике с решениямиэлементов по Уравнения по комбинаторике с решениями.

Заметим, что после определения вида соединения следует также выяснить, могут ли элементы в соединении повторяться, то есть выяснить, какую формулу необходимо использовать — для количества соединений без повторений или с повторениями.

Примеры решения задач:

Пример №42

На соревнования по легкой атлетике приехала команда из 12 спортсменок. Сколькими способами тренер может определить, кто из них побежит в эстафете 4 х 100 м на первом, втором, третьем и четвертом этапах?

Решение:

Уравнения по комбинаторике с решениямиКоличество способов выбрать из 12 спортсменок четырех для участия в эстафете равно количеству размещений из 12 элементов по 4 (без повторений), то есть Уравнения по комбинаторике с решениями

Для выбора формулы выясняем ответы на вопросы, приведенные выше. Поскольку для спортсменок важно, в каком порядке они будут бежать, то порядок при выборе элементов учитывается. В полученное соединение входят не все 12 заданных элементов. Следовательно, соответствующее соединение — размещение из 12 элементов по 4 (без повторений, поскольку каждая спортсменка может бежать только на одном этапе эстафеты).

Пример №43

Найдите количество трехзначных чисел, которые можно составить из цифр 1, 2, 3, 4, 5, 6, 7, если цифры в числе не повторяются.

Решение:

Уравнения по комбинаторике с решениямиКоличество трехзначных чисел, которые можно составить из семи цифр 1, 2, 3, 4, 5, 6, 7, равно числу размещений из 7 элементов по 3, то есть

Уравнения по комбинаторике с решениями

Для выбора формулы выясняем, что для чисел, которые мы будем составлять, порядок следования цифр учитывается и не все элементы выбираются (только 3 из заданных семи). Следовательно, соответствующее соединение — размещение из 7 элементов по 3 (без повторений).

Пример №44

Найдите количество трехзначных чисел, которые можно составить из цифр 1, 2, 3, 4, 5, 6, 0, если цифры в числе не повторяются.

Выбор формулы проводится таким же образом, как и в задаче 2. Следует учесть, что если число, составленное из трех цифр, начинается цифрой О, то оно не считается трехзначным. Следовательно, для ответов на вопросы задачи можно сначала из заданных 7 цифр записать все числа, состоящие из 3 цифр (см. пример 2), а затем из количества полученных чисел вычесть количество чисел, составленных из трех цифр, но начинающих цифрой 0. В последнем случае мы фактически будем из всех цифр без нуля (их 6) составлять двузначные числа. Тогда их количество равно числу размещений из 6 элементов по 2 (см. решение).

Также можно выполнить непосредственное вычисление, последовательно заполняя три места в трехзначном числе и используя правило произведения. В этом случае удобно сделать рассуждения наглядными, изображая соответствующие разряды в трехзначном числе в виде клеточек, например, так:

  • 6 возможностей
  • 6 возможностей
  • 5 возможностей

Решение:

Уравнения по комбинаторике с решениямиКоличество трехзначных чисел, которые можно составить из семи цифр (среди которых нет цифры 0), если цифры в числе не повторяются, равно числу размещений из 7 элементов по 3, то есть Уравнения по комбинаторике с решениями

Но среди данных цифр есть цифра 0, с которой не может начинаться трехзначное число. Поэтому из размещений из 7 элементов по 3 необходимо исключить те размещения, в которых первым элементом является цифра 0. Их количество равно числу размещений из 6 элементов по 2, то есть Уравнения по комбинаторике с решениямиСледовательно, искомое количество трехзначных чисел равноУравнения по комбинаторике с решениями

Пример №45

Решите уравнение Уравнения по комбинаторике с решениями

Решение:

Уравнения по комбинаторике с решениямиТогда получаем Уравнения по комбинаторике с решениямиНа ОДЗ это уравнение равносильно уравнениям:Уравнения по комбинаторике с решениями

Уравнения, в запись которых входят выражения, обозначающие количество соответствующих соединений из х элементов, считаются определенными только при натуральных значениях переменной х. В данном случае, чтобы выражение Уравнения по комбинаторике с решениямиимело смысл необходимо выбирать натуральные значения Уравнения по комбинаторике с решениями(в этом случае Уравнения по комбинаторике с решениямитакже существует и, конечно, Уравнения по комбинаторике с решениямиДля преобразования уравнения используем соответствующие формулы:Уравнения по комбинаторике с решениями

Перестановки

Перестановкой из п элементов называется любое упорядоченное множество из Уравнения по комбинаторике с решениямиэлементов

Напомним, что упорядоченное множество — это такое множество, для которого указано, какой элемент находится на первом месте, какой на втором. какой на Уравнения по комбинаторике с решениями

Например, переставляя цифры в числе 236 (там множество цифр уже упорядоченное), можно составить такие перестановки без повторений: (2; 3; 6), (2; 6; 3), (3; 2; 6), (3; 6; 2), (6; 2; 3), (6; 3; 2) — всего 6 перестановок*.

Количество перестановок без повторений из Уравнения по комбинаторике с решениямиэлементов обозначается Уравнения по комбинаторике с решениями(Р — первая буква французского слова permutation — перестановка). Как видим, Уравнения по комбинаторике с решениями

Уравнения по комбинаторике с решениямиФактически перестановки без повторений из Уравнения по комбинаторике с решениямиэлементов являются размещениями из Уравнения по комбинаторике с решениямиэлементов по Уравнения по комбинаторике с решениямибез повторений, поэтому Уравнения по комбинаторике с решениямиПроизведение 1 • 2 • 3 •. • Уравнения по комбинаторике с решениямиобозначается

Уравнения по комбинаторике с решениями!. Поэтому полученная формула числа перестановок без повторений из Уравнения по комбинаторике с решениямиэлементов может быть записана так:

Уравнения по комбинаторике с решениями

*Отметим, что каждая такая перестановка определяет трехзначное число, составленное из цифр 2,3,6 так, что цифры в числе не повторяются.

Например, Уравнения по комбинаторике с решениями(что совпадает с соответствующим значением, полученным выше).

С помощью факториалов формулу для числа размещений без повторений

Уравнения по комбинаторике с решениями

можно записать в другом виде. Для этого умножим и разделим выражение в формуле (1) на произведение Уравнения по комбинаторике с решениямиПолучаем Уравнения по комбинаторике с решениями

Следовательно, формула числа размещений без повторений из Уравнения по комбинаторике с решениямиэлементов по Уравнения по комбинаторике с решениямиможет быть записана так:

Уравнения по комбинаторике с решениями

Для того чтобы этой формулой можно было пользоваться при всех значениях Уравнения по комбинаторике с решениямив частности, при Уравнения по комбинаторике с решениямидоговорились считать, что

Уравнения по комбинаторике с решениями

Например, по формуле (2) Уравнения по комбинаторике с решениями

Обратим внимание, что в тех случаях, когда значение Уравнения по комбинаторике с решениями! оказывается очень большим, ответы оставляют записанными с помощью факториалов.

Например,Уравнения по комбинаторике с решениями

Примеры решения задач:

Напомним, что для выбора формулы при решении простейших комбинаторных задач достаточно выяснить следующее:

  • — Учитывается ли порядок следования элементов в соединении?
  • — Все ли заданные элементы входят в полученное соединение? Если, например, порядок следования элементов учитывается и все п заданных элементов используются в соединении, то по определению это перестановки из п элементов.

Пример №46

Найдите, сколькими способами можно восемь учащихся построить в колонну по одному.

Решение:

Уравнения по комбинаторике с решениямиКоличество способов равно числу перестановок из 8 элементов. То есть Уравнения по комбинаторике с решениями

Для выбора соответствующей формулы выясняем ответы на вопросы, приведенные выше. Поскольку порядок следования элементов учитывается и все 8 заданных элементов выбираются, то соответствующие соединения — это перестановки из 8 элементов без повторений. Их количество можно вычислить по формуле.

Пример №47

Найдите количество разных четырехзначных чисел, которые можно составить из цифр 0, 3, 7, 9 (цифры в числе не повторяются).

Решение:

Уравнения по комбинаторике с решениямиИз четырех цифр 0, 3, 7, 9, не повторяя заданные цифры, можно получить Уравнения по комбинаторике с решениямиперестановок. Перестановки, начинающиеся с цифры 0, не являются записью четырехзначного числа — их количество Уравнения по комбинаторике с решениями. Тогда искомое количество четырехзначных чисел равно

Уравнения по комбинаторике с решениями

Поскольку порядок следования элементов учитывается и для получения четырехзначного числа надо использовать все элементы, то искомые соединения — это перестановки из 4 элементов. Их количество — Уравнения по комбинаторике с решениями. При этом необходимо учесть, что в четырехзначном числе на первом месте не может стоять цифра 0. Таких чисел будет столько, сколько раз мы сможем выполнить перестановки из 3 оставшихся цифр, то есть Уравнения по комбинаторике с решениями.

Пример №48

Есть десять книг, из которых четыре — учебники. Сколькими способами можно поставить эти книги на полку так, чтобы все учебники стояли рядом?

Решение:

Уравнения по комбинаторике с решениямиСначала будем рассматривать учебники как одну книгу. Тогда на полке надо расставить не 10, а 7 книг. Это можно сделать Уравнения по комбинаторике с решениямиспособами. В каждом из полученных наборов книг можно выполнить еще Уравнения по комбинаторике с решениямиперестановок учебников. По правилу умножения искомое количество способов равно Уравнения по комбинаторике с решениями

Задачу можно решать в два этапа. На первом этапе условно будем считать все учебники за 1 книгу. Тогда получим 7 книг (6 не учебников + 1 условная книга — учебник). Порядок следования элементов учитывается и используются все элементы (поставить на полку необходимо все книги). Следовательно, соответствующие соединения — это перестановки из 7 элементов. Их количество — Уравнения по комбинаторике с решениями.

На втором этапе решения будем переставлять между собой только учебники. Это можно сделать Уравнения по комбинаторике с решениямиспособами. Поскольку нам надо переставить и учебники, и другие книги, то используем правило произведения.

Сочетания без повторений

Сочетанием без повторений из Уравнения по комбинаторике с решениямиэлементов по Уравнения по комбинаторике с решенияминазывается любое Уравнения по комбинаторике с решениями-элементное подмножество Уравнения по комбинаторике с решениями-элементного множества.

Например, из множества Уравнения по комбинаторике с решениями> можно составить следующие сочетания без повторений из трех элементов: Уравнения по комбинаторике с решениями

Количество сочетаний без повторений из п элементов по к элементов обозначается символом Уравнения по комбинаторике с решениями(читается: «Число сочетаний из Уравнения по комбинаторике с решениями» или «це из Уравнения по комбинаторике с решениями», С — первая буква французского слова combinaison — сочетание). Как видим,Уравнения по комбинаторике с решениями

Уравнения по комбинаторике с решениямиВыясним, сколько всего можно составить сочетаний без повторений из Уравнения по комбинаторике с решениямиэлементов по Уравнения по комбинаторике с решениями. Для этого используем известные нам формулы числа размещений и перестановок.

Составление размещения без повторений из Уравнения по комбинаторике с решениямиэлементов по Уравнения по комбинаторике с решениямипроведем в два этапа. Сначала выберем Уравнения по комбинаторике с решениямиразных элементов из заданного Уравнения по комбинаторике с решениями-элементного множества, не учитывая порядок выбора этих элементов (то есть выберем Уравнения по комбинаторике с решениями-элементное подмножество из Уравнения по комбинаторике с решениями-элементного множества — сочетание без повторений из Уравнения по комбинаторике с решениями-элементов по Уравнения по комбинаторике с решениями). По нашему обозначению это можно сделать Уравнения по комбинаторике с решениямиспособами. После этого полученное множество из к разных элементов упорядочим. Его можно упорядочить Уравнения по комбинаторике с решениямиспособами. Получим размещения без повторений из Уравнения по комбинаторике с решениямиэлементов по Уравнения по комбинаторике с решениями. Следовательно, количество размещений без повторений из Уравнения по комбинаторике с решениямиэлементов по Уравнения по комбинаторике с решениямив Уравнения по комбинаторике с решениямираз больше числа сочетаний без повторений из Уравнения по комбинаторике с решениямиэлементов по Уравнения по комбинаторике с решениями. То есть Уравнения по комбинаторике с решениямиОтсюда Уравнения по комбинаторике с решениямиУчитывая, что по формуле (2) Уравнения по комбинаторике с решениями, получаем Уравнения по комбинаторике с решениями

Например, Уравнения по комбинаторике с решениямисовпадает со значением, полученным выше.

Используя формулу (3), можно легко обосновать свойство 1 числа сочетаний без повторений, приведенное в таблице 21.

Уравнения по комбинаторике с решениями1) Поскольку Уравнения по комбинаторике с решениями

Для того чтобы формулу (4) можно было использовать и при Уравнения по комбинаторике с решениями, договорились считать, чтоУравнения по комбинаторике с решениями. Тогда по формуле (4) Уравнения по комбинаторике с решениями.

Если в формуле (3) сократить числитель и знаменатель наУравнения по комбинаторике с решениями, то получим формулу, по которой удобно вычислять Уравнения по комбинаторике с решениямипри малых значениях Уравнения по комбинаторике с решениями:

Уравнения по комбинаторике с решениями

Например, Уравнения по комбинаторике с решениями

Вычисление числа сочетаний без повторений с помощью треугольника Паскаля

Для вычисления числа сочетаний без повторений можно применять формулу (3): Уравнения по комбинаторике с решениями, а можно последовательно вычислять соответствующие значения, пользуясь таким свойством:

Уравнения по комбинаторике с решениями

Уравнения по комбинаторике с решениямиДля обоснования равенства (6) найдем сумму Уравнения по комбинаторике с решениямиучитывая, что Уравнения по комбинаторике с решениями

Уравнения по комбинаторике с решениями, следовательно,

Это равенство позволяет последовательно вычислять значения Уравнения по комбинаторике с решениямис помощью специальной таблицы, которая называется треугольником Паскаля. Если считать, что Уравнения по комбинаторике с решениями, то таблица будет иметь следующий вид (табл. 23).

Каждая строка этой таблицы начинается с единицы и заканчивается единицей Уравнения по комбинаторике с решениями.

Если какая-либо строка уже заполнена, например, третья, то в четвертой строке надо записать на первом месте единицу. На втором месте запишем число, равное сумме двух чисел третьей строки, стоящих над ним левее и правее (поскольку по формуле (6)Уравнения по комбинаторике с решениями.

Уравнения по комбинаторике с решениями

На третьем месте запишем число, равное сумме двух следующих чисел третьей строки, стоящих над ним левее и правееУравнения по комбинаторике с решениями, и т. д. (а на последнем месте снова запишем единицу).

Примеры решения задач:

Обратим внимание, что, как и раньше, для выбора формулы при решении простейших комбинаторных задач достаточно ответить на вопросы:

  1. Учитывается ли порядок следования элементов в соединении?
  2. Все ли заданные элементы входят в полученное соединение?

Для выяснения того, что заданное соединение является сочетанием, достаточно ответить только на первый вопрос. Если порядок следования элементов не учитывается, то по определению это сочетания из Уравнения по комбинаторике с решениямиэлементов по Уравнения по комбинаторике с решениямиэлементов.

Пример №49

Из 12 членов туристической группы надо выбрать трех дежурных. Сколькими способами можно сделать этот выбор?

Решение:

Уравнения по комбинаторике с решениямиКоличество способов выбрать из 12 туристов трех дежурных равно количеству сочетаний из 12 элементов по 3 (без повторений), то есть

Уравнения по комбинаторике с решениями

Для выбора соответствующей формулы выясняем ответы на вопросы, приведенные выше. Поскольку порядок следования элементов не учитывается (для дежурных неважно, в каком порядке их выберут), то соответствующее соединение является сочетанием из 12 элементов по 3 (без повторений). Для вычисления можно использовать формулы (3) или (5), в данном случае применяем формулу (3):Уравнения по комбинаторике с решениями

Пример №50

Из вазы с фруктами, в которой лежит 10 разных яблок и 5 разных груш, требуется выбрать 2 яблока и 3 груши. Сколькими способами можно сделать такой выбор?

Решение:

Уравнения по комбинаторике с решениямиВыбрать 2 яблока из 10 можно Уравнения по комбинаторике с решениямиспособами. При каждом выборе яблок груши можно выбрать способами. Тогда по правилу произведения выбор требуемых фруктов можно выполнить Уравнения по комбинаторике с решениямиспособами. Получаем

Уравнения по комбинаторике с решениями

Сначала отдельно выберем 2 яблока из 10 и 3 груши из 5. Поскольку при выборе яблок или груш порядок следования элементов не учитывается, то соответствующие соединения — сочетания без повторений.

Учитывая, что требуется выбрать 2 яблока и 3 груши, используем правило произведения и перемножим полученные возможности выбора яблок(Уравнения по комбинаторике с решениями) и груш (Уравнения по комбинаторике с решениями).

Бином Ньютона

Уравнения по комбинаторике с решениями

Поскольку Уравнения по комбинаторике с решениямито формулу бинома Ньютона можно записать еще и так:

Уравнения по комбинаторике с решениями

Общий член разложения степени бинома имеет вид Уравнения по комбинаторике с решениями

Коэффициенты Уравнения по комбинаторике с решенияминазывают биномиальными коэффициентами.

Свойства биномиальных коэффициентов:

  1. Число биномиальных коэффициентов (а следовательно, и число слагаемых в разложении Уравнения по комбинаторике с решениямистепени бинома) равноУравнения по комбинаторике с решениями
  2. Коэффициенты членов, равноудаленных от начала и конца разложения, равны между собой (поскольку Уравнения по комбинаторике с решениями
  3. Сумма всех биномиальных коэффициентов равна Уравнения по комбинаторике с решениямиУравнения по комбинаторике с решениями
  4. Сумма биномиальных коэффициентов, стоящих на четных местах, равна сумме биномиальных коэффициентов, стоящих на нечетных местах.
  5. Для вычисления биномиальных коэффициентов можно воспользоваться треугольником Паскаля, в котором вычисления коэффициентов основываются на формуле Уравнения по комбинаторике с решениями

Уравнения по комбинаторике с решениями

Уравнения по комбинаторике с решениями

В каждом ряду по краям стоят единицы, а каждое из остальных чисел равно сумме двух чисел, находящихся над ним справа и слева Например, Уравнения по комбинаторике с решениями

Объяснение и обоснование Бинома Ньютона

Двучлен вида а + х также называют биномом. Из курса алгебры известно, что: Уравнения по комбинаторике с решениями

Можно заметить, что коэффициенты разложения степени бинома Уравнения по комбинаторике с решениямипри Уравнения по комбинаторике с решениямисовпадают с числами в соответствующей строке треугольника Паскаля. Оказывается, что это свойство выполняется для любого натурального Уравнения по комбинаторике с решениямито есть справедлива формула:

Уравнения по комбинаторике с решениями

Формулу (7) называют биномом Ньютона. Правая часть этого равенства называется разложением степени бинома Уравнения по комбинаторике с решениямиУравнения по комбинаторике с решенияминазывают биномиальными коэффициентами. Общий член разложения степени бинома имеет вид Уравнения по комбинаторике с решениями

Уравнения по комбинаторике с решениямиОбосновать формулу (7) можно, например, следующим образом.

Если раскрыть скобки в выражении Уравнения по комбинаторике с решениямито есть умножить бином а + х сам на себя Уравнения по комбинаторике с решениямираз, то получим многочлен Уравнения по комбинаторике с решениямистепени относительно переменной х. Тогда результат можно записать так:

Уравнения по комбинаторике с решениями

Чтобы найти значение Уравнения по комбинаторике с решениямиподставим в обе части равенства (8) вместо х значение 0. Получаем Уравнения по комбинаторике с решениямиможем записать:

Уравнения по комбинаторике с решениями

Чтобы найти Уравнения по комбинаторике с решениямисначала возьмем производную от обеих частей равенства (8):

Уравнения по комбинаторике с решениями

затем, подставив в обе части полученного равенства (9) х = 0, получим: Уравнения по комбинаторике с решениямиУчитывая, чтоУравнения по комбинаторике с решениямиможем записать: Уравнения по комбинаторике с решениямиАналогично, чтобы найти Уравнения по комбинаторике с решениямивозьмем производную от обеих частей равенства (9):

Уравнения по комбинаторике с решениями

и, подставив х = 0 в равенство (10), получим Уравнения по комбинаторике с решениямиТогда Уравнения по комбинаторике с решениямиДругие коэффициенты находят аналогично. Если продифференцировать Уравнения по комбинаторике с решениямираз равенство (8), то получим:

Уравнения по комбинаторике с решениями

Подставляя в последнее равенство х = 0, имеем

Уравнения по комбинаторике с решениями

Уравнения по комбинаторике с решениями

В каждом ряду по краям стоят единицы, а каждое из остальных чисел равно сумме двух чисел, находящихся над ним справа и слева

Умножим обе части равенства (11) на Уравнения по комбинаторике с решениямии найдем коэффициент

Уравнения по комбинаторике с решениями. Подставляя найденные значения Уравнения по комбинаторике с решениями

1, 2, . Уравнения по комбинаторике с решениями) в равенство (8), получаем равенство (7).Уравнения по комбинаторике с решениями

Записывая степень двучлена по формуле бинома Ньютона для небольших значений п, биномиальные коэффициенты можно вычислять по треугольнику Паскаля (табл. 25, см. также табл. 24).

Например,Уравнения по комбинаторике с решениями

Так как Уравнения по комбинаторике с решениямиформулу бинома Ньютона можно записать в виде:

Уравнения по комбинаторике с решениями

а учитывая, чтоУравнения по комбинаторике с решениями, еще и так:

Уравнения по комбинаторике с решениями

Если в формуле бинома Ньютона (12) заменить х на (-х), то получим формулу возведения в степень разности а — х:

Уравнения по комбинаторике с решениями. Например, ( Уравнения по комбинаторике с решениями(знаки членов разложения чередуются!).

Свойства биномиальных коэффициентов

1. Число биномиальных коэффициентов (а следовательно, и число слагаемых) в разложении Уравнения по комбинаторике с решениями-й степени бинома равно Уравнения по комбинаторике с решениями+ 1, поскольку разложение содержит все степени х от 0 до Уравнения по комбинаторике с решениями(и других слагаемых не содержит).

2. Коэффициенты членов, равноудаленных от начала и конца разложения, равны между собой, посколькуУравнения по комбинаторике с решениями

3. Сумма всех биномиальных коэффициентов равна 2″.

Уравнения по комбинаторике с решениямиДля обоснования полагаем в равенстве (13) (или в равенстве (7)) значения а = х = 1 и получаем Уравнения по комбинаторике с решениями

Например, Уравнения по комбинаторике с решениями

4. Сумма биномиальных коэффициентов, стоящих на четных местах, равна сумме биномиальных коэффициентов, стоящих на нечетных местах,

Уравнения по комбинаторике с решениямиДля обоснования возьмем в равенстве (13) значения а =1, х = —1. Получаем

Уравнения по комбинаторике с решениями

Тогда Уравнения по комбинаторике с решениями

Примеры решения задач:

Пример №51

По формуле бинома Ньютона найдите разложение степени Уравнения по комбинаторике с решениями

Для нахождения коэффициентов разложения можно использовать треугольник Паскаля или вычислять их по общей формуле. По треугольнику Паскаля коэффициенты равны: 1, 6, 15, 20, 15, б, 1. Учитывая, что при возведении в степень разности знаки членов разложения чередуются, получаем

Уравнения по комбинаторике с решениямиДля упрощения записи ответа можно избавиться от иррациональности в знаменателях полученных выражений (см. решение) или сначала учесть, что ОДЗ заданного выражения: х > 0, и тогда Уравнения по комбинаторике с решениямиТо есть заданное выражение можно записать так: Уравнения по комбинаторике с решениямии возвести в степень последнее выражение.

Решение:

Уравнения по комбинаторике с решениями

Пример №52

В разложении степени Уравнения по комбинаторике с решенияминайти член, содержащий Уравнения по комбинаторике с решениями

Решение:

► ОДЗ: Уравнения по комбинаторике с решениями> 0. ТогдаУравнения по комбинаторике с решениями

Общий член разложения: Уравнения по комбинаторике с решениями

По условию член разложения должен содержатьУравнения по комбинаторике с решениями, следовательно,

Уравнения по комбинаторике с решениями. Отсюда Уравнения по комбинаторике с решениями

Тогда член разложения, содержащий Уравнения по комбинаторике с решениями, равенУравнения по комбинаторике с решениями

На ОДЗ (b > 0) каждое слагаемое в заданном двучлене можно записать как степень с дробным показателем. Это позволит проще записать общий член разложения степениУравнения по комбинаторике с решениями: Уравнения по комбинаторике с решениями(где Уравнения по комбинаторике с решениями= 0, 1, 2, . Уравнения по комбинаторике с решениями), выяснить, какой из членов разложения содержит Уравнения по комбинаторике с решениями, и записать его.

Чтобы упростить запись общего члена разложения, удобно отметить, чтоУравнения по комбинаторике с решениями

Видео:9 класс, 26 урок, Комбинаторные задачиСкачать

9 класс, 26 урок, Комбинаторные задачи

Зачем нужна комбинаторика

Для решения задач с использованием классического определения вероятности необходимо знать основные правила и формулы комбинаторики -раздела математики, изучающего методы решения комбинаторных задач — т.е. задач, связанных с подсчетом числа различных комбинаций.

Пусть Уравнения по комбинаторике с решениями— элементы конечного множества. Сформулируем два важных правила, часто применяемых при решении комбинаторных задач.

Правило суммы

Если элемент Уравнения по комбинаторике с решениямиможет быть выбран Уравнения по комбинаторике с решениямиспособами, элемент / Уравнения по комбинаторике с решениямиспособами, . элемент Уравнения по комбинаторике с решениямиспособами, то выбор одного из элементов Уравнения по комбинаторике с решениямиможет быть осуществлен пУравнения по комбинаторике с решениямиспособами.

Пример №53

В группе 30 студентов. Известно, что 5 из них на экзамене по математике получили оценку «отлично», 10 — оценку «хорошо», остальные -«удовлетворительно». Сколько существует способов выбрать одного студента, получившего на экзамене оценку «отлично» или «хорошо»?

Решение:

Студент, получивший оценку «отлично» может быть выбранУравнения по комбинаторике с решениямиспособами, оценку «хорошо» — Уравнения по комбинаторике с решениямиспособами. По правилу суммы существует Уравнения по комбинаторике с решениямиспособов выбора одного студента, получившего на экзамене оценку «отлично» или «хорошо». Уравнения по комбинаторике с решениями

Правило произведения

Если элемент Уравнения по комбинаторике с решениямиможет быть выбран Уравнения по комбинаторике с решениямиспособами, после этого элемент Уравнения по комбинаторике с решениямиможет быть выбран Уравнения по комбинаторике с решениямиспособами после каждого такого выбора элемент Уравнения по комбинаторике с решениямиможет быть выбран Уравнения по комбинаторике с решениямиспособами, то выбор всех элементов Уравнения по комбинаторике с решениямив указанном порядке может быть осуществлен Уравнения по комбинаторике с решениямиспособами.

Пример №54

В группе 30 студентов. Необходимо выбрать старосту, его заместителя и профорга. Сколько существует способов это сделать?

Решение:

Старостой может быть выбран любой из 30 студентов, его заместителем – любой из оставшихся 29, а профоргом – любой из оставшихся 28 студентов, т.е. Уравнения по комбинаторике с решениямиПо правилу произведения общее число способов выбора старосты, его заместителя и профорга равно Уравнения по комбинаторике с решениями= = 24360 способов. ◄

Пусть дано множество из n различных элементов. Из этого множества могут быть образованы подмножества из m элементов (0 ≤ m ≤n). Например, из 5 элементов a, b, c, d, e могут быть отобраны комбинации по 2 элемента – ab, bc, cd, ba и т.д., по 3 элемента – abc, cbd, cba и т.д.

Если комбинации из n элементов по m отличаются либо составом элементов, либо порядком их расположения (либо и тем и другим), то такие комбинации называют размещениями из n элементов по m. Число размещений из n элементов по m находится по формуле Уравнения по комбинаторике с решениямигде n! равно произведению n первых чисел натурального ряда, т.е. n! = 1·2·…·n.

Пример №55

Сколько можно записать двузначных чисел, используя без повторения цифры от 1 до 5?

Решение:

В данном случае двузначное число является комбинацией из пяти цифр по две цифры. Поскольку числа отличаются как составом входящих в них цифр, так и порядком их расположения, то в данном случае двузначные числа являются размещениями из пяти цифр по две. Число таких размещений

Уравнения по комбинаторике с решениямиЕсли комбинации из n элементов по m отличаются только с о с т а в о м элементов (порядок их расположения не имеет значения), то такие комбинации называют сочетаниями из n элементов по m.

Число сочетаний из n элементов по m находится по формуле Уравнения по комбинаторике с решениями

Пример №56

Необходимо выбрать в подарок две из пяти имеющихся различных книг. Сколькими способами можно это сделать?

Решение:

Из смысла задачи следует, что порядок выбора книг не имеет значения. Здесь важен только их состав. Поэтому в данном случае комбинации книг представляют собой сочетания из 5 книг по 2. Число таких комбинаций Уравнения по комбинаторике с решениямиЕсли в размещениях из n элементов по m некоторые из элементов (или все) могут оказаться одинаковыми, то такие размещения называют размещениями с повторениями из n элементов по m. Число размещений с повторениями равно Уравнения по комбинаторике с решениями

Пример №57

Сколько можно записать трехзначных чисел, которые не содержат цифр 0 и 5?

Решение:

В данном случае трехзначное число является комбинацией из восьми цифр (0 и 5 не учитываются) по три цифры. При этом некоторые из цифр (или все) могут повторяться. Поэтому в данном случае трехзначные числа является размещениями с повторениями из восьми цифр по три. Число таких размещений с повторениями Уравнения по комбинаторике с решениямиЕсли в сочетаниях из n элементов по m некоторые из элементов (или все) могут оказаться одинаковыми, то такие сочетания называют сочетаниями с повторениями из n элементов по m. Число сочетаний с повторениями равно Уравнения по комбинаторике с решениямигде Уравнения по комбинаторике с решениямиопределяется по формуле (1.6).

Пример №58

В почтовом отделении продаются открытки восьми видов. Сколькими способами можно купить в нем три открытки?

Решение:

Учитывая, что порядок выбора открыток не имеет значения, а важен только их состав, причем некоторые из открыток (или все) могут оказаться одинаковыми, искомое число способов находим по формуле числа сочетаний с повторениями Уравнения по комбинаторике с решениямиЕсли комбинации из n элементов отличаются только порядком расположения элементов, то такие комбинации называют перестановками из n элементов. Число перестановок из n элементов находится по формуле Уравнения по комбинаторике с решениями

Пример №59

Порядок выступления 5 участников конкурса определяется жребием. Сколько различных вариантов жеребьевки при этом возможно?

Решение:

Каждый вариант жеребьевки отличается только порядком участников конкурса, т.е. является перестановкой из 5 элементов. Их число равно Уравнения по комбинаторике с решениямиЕсли в перестановках из общего числа n элементов есть k различных элементов, при этом 1-й элемент повторяется Уравнения по комбинаторике с решениямираз, 2-й элемент – Уравнения по комбинаторике с решениямираз, k-й элемент – Уравнения по комбинаторике с решениямираз, причемУравнения по комбинаторике с решениями, то такие перестановки называют перестановками с повторениями из n элементов. Число перестановок с повторениями равно Уравнения по комбинаторике с решениями

Пример №60

Сколько можно составить шестизначных чисел, состоящих из цифр 3, 5, 7, в которых цифра 3 повторяется 3 раза, цифра 5 – 2 раза, цифра 7 – 1 раз?

Решение:

Каждое шестизначное число отличается от другого порядком следования цифр (причем Уравнения по комбинаторике с решениямиа их сумма равна 6), т.е. является перестановкой с повторениями из 6 элементов. Их число равно

Уравнения по комбинаторике с решениями

Рекомендую подробно изучить предметы:
  1. Теория вероятностей
  2. Математическая статистика
Ещё лекции с примерами решения и объяснением:
  • Классическое определение вероятности
  • Геометрические вероятности
  • Теоремы сложения и умножения вероятностей
  • Формула полной вероятности
  • Математическая обработка динамических рядов
  • Корреляция — определение и вычисление
  • Элементы теории ошибок
  • Методы математической статистики

При копировании любых материалов с сайта evkova.org обязательна активная ссылка на сайт www.evkova.org

Сайт создан коллективом преподавателей на некоммерческой основе для дополнительного образования молодежи

Сайт пишется, поддерживается и управляется коллективом преподавателей

Whatsapp и логотип whatsapp являются товарными знаками корпорации WhatsApp LLC.

Cайт носит информационный характер и ни при каких условиях не является публичной офертой, которая определяется положениями статьи 437 Гражданского кодекса РФ. Анна Евкова не оказывает никаких услуг.

Видео:Комбинаторика. Сочетание. 10 класс.Скачать

Комбинаторика. Сочетание. 10 класс.

Конспект урока на тему «Решение комбинаторных уравнений» (10 класс)

Сочетаниями без повторений занимался еще великий Паскаль. Он предложил специальную таблицу значений сочетаний без повторений.

Значения представлены в табл. которая называется треугольником Паскаля.

Этот треугольник удивительно красив своей математической красотой, и в его числах можно при желании отыскать различные закономерности. Его можно представить несколько иначе – в виде [26]: равнобедренного треугольника (рис. 10).

Уравнения по комбинаторике с решениями

Рис. 10. Треугольник Паскаля

Здесь каждое число, кроме единиц на боковых сторонах, является суммой двух чисел, стоящих над ним. Поэтому:

(приводим к общему знаменателю)

(выносим n ! за скобку в знаменателе)

Из этого соотношения и вытекает эффективный способ рекуррентного вычисления значений биномиальных коэффициентов.

Докажем соотношение 1)

Это может использоваться при вычислениях, например, вместо можно вычислить .

Докажем соотношение 2)

Имеется формула, называемая биномом Ньютона, которая использует выражения числа сочетаний с повторениями

где а, b – действительные или комплексные числа.

Коэффициенты называются биномиальными.

Докажем формулу бинома Ньютона по индукции. Доказательство по индукции предполагает:

1) базис индукции – доказательство того, что формула верна для конкретного n , например, для n =1. В нашем случае мы убедились, что формула верна для n =2,3,4. Убедимся, что она верна и для n =1.

2) индукционный шаг. Предполагая, что формула верна для некоторого n , убеждаются, что тогда она верна и для n +1.

3) при истинности шагов 1 и 2 заключают, что формула верна для любого n .

Приступим к индукционному шагу.

Возьмем выражение и получим из него выражение для n +1. Очевидно, что это можно сделать путем умножения на a + b :

Преобразуем полученное выражение:

Для выполнения индукционного шага необходимо показать, что это выражение равно выражению:

Рассмотрим подвыражение выражения (1): и заменим i на i -1.

Получим , т.е. одинаковые коэффициенты перед выражениями , для числа сочетаний в первом и втором подвыражении выражения (1).Это позволит вынести за скобку. Но тогда в не учтен n -й член подвыражения (суммирование идет до n ): тогда, учитывая его, получаем:

Нетрудно видеть, что можно заменить на , кроме того, мы уже доказали, что , поэтому: , что, очевидно, равно выражению:

По индукции получаем, что формула бинома Ньютона верна для любого n .

С использованием бинома Ньютона докажем следствие №1 о количестве подмножеств множества из n элементов:

Рассмотрим следствие №2: .

На использовании бинома Ньютона основано понятие производящей функции – функции, позволяющей получать комбинаторные числа без вычисления факториала:

. Здесь – функция, производящая биномиальные коэффициенты.

При n =1 получаем 1+ x , т.е. (коэффициент перед 1), (коэффициент перед x ).

При n =2 получаем (1+ x ) 2 =1+2 x + x 2 , т.е. и т.д.

Решение комбинаторных уравнений

В комбинаторике тоже могут решаться уравнения, особенностью которых является то, что неизвестная принадлежит множеству натуральных чисел. Например, уравнения вида , xN , где N – множество натуральных чисел или вида:

При решении комбинаторных уравнений часто необходимо уметь выполнять действия с факториалами типа:

Например, в задаче о сравнении пар записей в базе данных из n записей:

, – что и требовалось доказать.

В комбинаторике рассматриваются и другие типовые комбинаторные комбинации, например, разбиения n -элементного множества на k подмножеств, которые называются блоками разбиения. В информатике вычисления на конечных математических структурах часто называют комбинаторными вычислениями, и они требуют комбинаторного анализа для установления свойств и оценки применимости используемых алгоритмов. На рис. 11 приведен один из возможных вариантов классификации основных комбинаций.

Уравнения по комбинаторике с решениями

Рис. 11. Основные комбинации

Комбинаторные задачи могут быть решены, например, системой компьютерной математики Matematica (3,4) фирмы Wolfram Research , Inc . – пакет расширения «Дискретная математика» ( DiscreteMath ) – комбинаторика и ее функции ( Combinatorica , CombinatorialFunctions ): функции перестановок и сочетаний и др.

Пример 1. Решить уравнение

Уравнения по комбинаторике с решениями

Уравнения по комбинаторике с решениями

и представим правую часть в виде

Уравнения по комбинаторике с решениями,

Уравнения по комбинаторике с решениями

Уравнения по комбинаторике с решениями

Уравнения по комбинаторике с решениямиоткуда следует

Уравнения по комбинаторике с решениями

Уравнения по комбинаторике с решениями

x + 3 = 11 и x = 8.

Пример 2. Решить уравнение

Уравнения по комбинаторике с решениями

Решение. По условию x – целое число, удовлетворяющее неравенством Уравнения по комбинаторике с решениямиПерепишем уравнение в виде

Уравнения по комбинаторике с решениями

Уравнения по комбинаторике с решениями

Уравнения по комбинаторике с решениями

Уравнения по комбинаторике с решениями

Уравнения по комбинаторике с решениями

Уравнения по комбинаторике с решениями

откуда, после упрощений, получаем

Уравнения по комбинаторике с решениями

Уравнения по комбинаторике с решениями> 4

Пример 3. Решить систему уравнений

Уравнения по комбинаторике с решениями

Решение. Из второго уравнение находим

Уравнения по комбинаторике с решениямиРешая последнее уравнение, получаем Уравнения по комбинаторике с решениямиНо так как Уравнения по комбинаторике с решениямине пригодно к решению уравнения, значит x = 18.

Подставляя x = 18 в первое уравнение системы, найдем

Уравнения по комбинаторике с решениями

Уравнения по комбинаторике с решениями

18 – y = y + 2, y = 8.

Итак, x = 18, y = 8.

Пример 4. Решить систему уравнений

Уравнения по комбинаторике с решениями

Решение. Перепишем систему уравнений в виде

Уравнения по комбинаторике с решениямиили, после упрощений получим

Уравнения по комбинаторике с решениямиоткуда следует x = 2, y = 6.

Решите уравнение (22–25) .

1)Уравнения по комбинаторике с решениями=42;

ОДЗ: хУравнения по комбинаторике с решениямиN; x > 2

Уравнения по комбинаторике с решениями= 42

Уравнения по комбинаторике с решениями

Уравнения по комбинаторике с решениями

Уравнения по комбинаторике с решениями=-6( исключить – не входит в ОДЗ); Уравнения по комбинаторике с решениями=7

Уравнения по комбинаторике с решениями=56х;

ОДЗ: хУравнения по комбинаторике с решениямиN; x > 3

Уравнения по комбинаторике с решениями= Уравнения по комбинаторике с решениями

Уравнения по комбинаторике с решениями

(Уравнения по комбинаторике с решениями

Уравнения по комбинаторике с решениями((Уравнения по комбинаторике с решениями

Уравнения по комбинаторике с решениямиили Уравнения по комбинаторике с решениями-3Уравнения по комбинаторике с решениями

Уравнения по комбинаторике с решениями1 =0(исключить) или х 2 =-6 (исключить); х 3 =9 (входит в ОДЗ).

3)Уравнения по комбинаторике с решениями=30;

ОДЗ: хУравнения по комбинаторике с решениямиN; x+1 > 2; х > 1

Уравнения по комбинаторике с решениями= Уравнения по комбинаторике с решениями

Уравнения по комбинаторике с решениями

Уравнения по комбинаторике с решениями

Уравнения по комбинаторике с решениями=-6( исключить – не входит в ОДЗ); Уравнения по комбинаторике с решениями=5.

4) 5Уравнения по комбинаторике с решениями=Уравнения по комбинаторике с решениями;

ОДЗ: Уравнения по комбинаторике с решениями Уравнения по комбинаторике с решениямихУравнения по комбинаторике с решениями

Уравнения по комбинаторике с решениями; Уравнения по комбинаторике с решениями=Уравнения по комбинаторике с решениями

Уравнения по комбинаторике с решениями= Уравнения по комбинаторике с решениями

Уравнения по комбинаторике с решениями= Уравнения по комбинаторике с решениями

(20(х-2)-(х+1)(х+2))Уравнения по комбинаторике с решениямихУравнения по комбинаторике с решениями

(20х-40-х 2 +2х+х+2)=0 или х=0 или х-1=0

х 2 +3х-20х+42=0 х 1 =0 х 2 =1

х 2 -17х+42=0 корни 0 и 1 не входят в ОДЗ

Уравнения по комбинаторике с решениями= 21 ОДЗ: хУравнения по комбинаторике с решениямиN; x-3 > 2 ; x > 3

Уравнения по комбинаторике с решениями= Уравнения по комбинаторике с решениями

Уравнения по комбинаторике с решениями

Уравнения по комбинаторике с решениями

Уравнения по комбинаторике с решениями— 7х + 12 – 42 = 0

Уравнения по комбинаторике с решениями— 7х – 30 = 0

х 1 =10 х 2 = — 3 (не входит в ОДЗ)

2) Уравнения по комбинаторике с решениями; ОДЗ: хУравнения по комбинаторике с решениямиN; x > 3

Уравнения по комбинаторике с решениями= Уравнения по комбинаторике с решениями

Уравнения по комбинаторике с решениями= Уравнения по комбинаторике с решениями

4х(х-2)(х-1) = 6Уравнения по комбинаторике с решениями

х(4х 2 – 12х+8-30х+90)=0

х=0 или 4х 2 – 42х + 98 = 0

2х 2 – 21х + 49 = 0

Уравнения по комбинаторике с решениями= 15(х-1) ОДЗ: хУравнения по комбинаторике с решениямиN; x > 3

Уравнения по комбинаторике с решениями= 15(х-1)

Уравнения по комбинаторике с решениями= (х-1)х х 1 = 0 или х 2 = 1 — не входят в ОДЗ

Уравнения по комбинаторике с решениями= Уравнения по комбинаторике с решениямиОДЗ: хУравнения по комбинаторике с решениямиN; x > 4

Уравнения по комбинаторике с решениями= Уравнения по комбинаторике с решениями

4(х-2)! = 24Уравнения по комбинаторике с решениями

х 1 =12; х 2 = — 7(не входит в ОДЗ)

Уравнения по комбинаторике с решениями= 43 ОДЗ: хУравнения по комбинаторике с решениямиN; x > 5

Уравнения по комбинаторике с решениями= 43

Уравнения по комбинаторике с решениями

х 1 =10; х 2 = 3 (не входит в ОДЗ)

Уравнения по комбинаторике с решениями= 89 ОДЗ: хУравнения по комбинаторике с решениямиN; x > 7

Уравнения по комбинаторике с решениями

х 2 – 11х – 60 = 0

х 1 =15; х 2 = — 4(не входит в ОДЗ)

Уравнения по комбинаторике с решениями+ Уравнения по комбинаторике с решениями= 162 ОДЗ: хУравнения по комбинаторике с решениямиN; x > 1

Уравнения по комбинаторике с решениями= 162

Уравнения по комбинаторике с решениями= 162

2Уравнения по комбинаторике с решениями

24х + х 2 + 7х + 12 – 324 = 0

х 2 + 31х – 312 = 0

х 1 =8; х 2 = — 39(не входит в ОДЗ)

Уравнения по комбинаторике с решениями= Уравнения по комбинаторике с решениями

ОДЗ: Уравнения по комбинаторике с решениямиx > 4

Уравнения по комбинаторике с решениями= Уравнения по комбинаторике с решениями

Уравнения по комбинаторике с решениями= Уравнения по комбинаторике с решениями

(х-2)(х-1)х = 0 или (х-3)-45 = 0

х 1 =2; х 2 = 1 х 3 =0 — не входят в ОДЗ х 4 = 48

Уравнения по комбинаторике с решениями= 42 ОДЗ: хУравнения по комбинаторике с решениямиN; x > 4

Уравнения по комбинаторике с решениями= 12

Уравнения по комбинаторике с решениями= 12 х 2 – х – 12 = 0 х 1 =4; х 2 = — 3(не входит в ОДЗ) Ответ: 4.

Уравнения по комбинаторике с решениями= 90 ОДЗ: Уравнения по комбинаторике с решениями Уравнения по комбинаторике с решениямиУравнения по комбинаторике с решениями

Уравнения по комбинаторике с решениями= 90

х 1 =10; х 2 = — 9(не входит в ОДЗ)

Уравнения по комбинаторике с решениями= 132 ОДЗ: Уравнения по комбинаторике с решениями

Уравнения по комбинаторике с решениями= 132

Уравнения по комбинаторике с решениями= 132

x 2 +3 x +2–132 = 0

х 1 =10; х 2 = — 13(не входит в ОДЗ)

Уравнения по комбинаторике с решениями= 110 ОДЗ: Уравнения по комбинаторике с решениями

Уравнения по комбинаторике с решениями= 110

Уравнения по комбинаторике с решениями= 110

x 2 +3 x +2– 110 = 0

x 2 +3 x – 108 = 0

х 1 =9; х 2 = — 12(не входит в ОДЗ)

Уравнения по комбинаторике с решениямиОДЗ: Уравнения по комбинаторике с решениями

Уравнения по комбинаторике с решениями

Уравнения по комбинаторике с решениями

Уравнения по комбинаторике с решениями Уравнения по комбинаторике с решениямиУравнения по комбинаторике с решениями

Уравнения по комбинаторике с решениямирешаем методом сложения — 5у = -30; у = 6

Уравнения по комбинаторике с решениямиОДЗ: Уравнения по комбинаторике с решениямиУравнения по комбинаторике с решениями; уУравнения по комбинаторике с решениями

Уравнения по комбинаторике с решениями Уравнения по комбинаторике с решениямиУравнения по комбинаторике с решениями

Уравнения по комбинаторике с решениями Уравнения по комбинаторике с решениямиУравнения по комбинаторике с решениями

Уравнения по комбинаторике с решениями

(х-3)(х-2)(х-1) = 3Уравнения по комбинаторике с решениями

4) Уравнения по комбинаторике с решениями

Уравнения по комбинаторике с решениями

Уравнения по комбинаторике с решениями

Уравнения по комбинаторике с решениями

Уравнения по комбинаторике с решениями

Сколько двузначных чисел можно составить из цифр 1. 3, 5, 8, 9 так, чтобы в каждом числе не было одинаковых цифр?

Из 6 открыток надо выбрать 3. Сколькими способами это можно сделать?

💡 Видео

Андрей Райгородский | «Комбинаторные задачи»Скачать

Андрей Райгородский | «Комбинаторные задачи»

Cистемы уравнений. Разбор задания 6 и 21 из ОГЭ. | МатематикаСкачать

Cистемы уравнений. Разбор задания 6 и 21 из ОГЭ.  | Математика

Решение биквадратных уравнений. 8 класс.Скачать

Решение биквадратных уравнений. 8 класс.

Комбинаторика. Основные формулы (перестановки, сочетания, размещения) и примеры решения задач.Скачать

Комбинаторика. Основные формулы (перестановки, сочетания, размещения) и примеры решения задач.

Неравенства с двумя переменными. 9 класс.Скачать

Неравенства с двумя переменными. 9 класс.

Решение тригонометрических уравнений. Подготовка к ЕГЭ | Математика TutorOnlineСкачать

Решение тригонометрических уравнений. Подготовка к ЕГЭ | Математика TutorOnline

Комбинаторика. Комбинаторные задачи. 10 класс.Скачать

Комбинаторика. Комбинаторные задачи. 10 класс.

Уравнение, комбинаторика, сочетания, факториалы | Это how? #5Скачать

Уравнение, комбинаторика, сочетания, факториалы | Это how? #5

Как распознать талантливого математикаСкачать

Как распознать талантливого математика

Решение задач по теории вероятностей | Часть 1Скачать

Решение задач по теории вероятностей | Часть 1

✓ Комбинаторика. Свойства чисел сочетаний | Ботай со мной #132 | Борис ТрушинСкачать

✓ Комбинаторика. Свойства чисел сочетаний | Ботай со мной #132 | Борис Трушин
Поделиться или сохранить к себе: